Section 2 Flashcards

1
Q

A 33-year-old African-American female with a history of intravenous drug use and alcohol use presented to the emergency department with eye discharge, painful eye movement, eye swelling on the right side and double vision for two days. The patient reported having a fall two days ago, but she was not able to remember the details. Vitals showed blood pressure 160/95 mmHg, heart rate 105 bpm, and temperature 37.5 C. She is alert and oriented x3. There is chemosis, periorbital edema, proptosis, mild restriction of all extraocular movements of the right eye and mild drooping of the right eyelid. The visual acuity was 20/20 normal on both eyes. A bruit was noticed over the right orbit. Pupillary reflexes were normal. Fundus exam showed mildly dilated vessels but no papilledema. There was a small bruise on the right side of the face on the maxillary area. The rest of the exam was unremarkable. Basic blood workup came back unremarkable. CT head without contrast showed a small maxillary fracture, proptosis, and orbital edema on the right side. What are the most likely diagnosis and the best evaluation for the definitive diagnosis?

A
  1. Cavernous sinus tumor so perform a biopsy
  2. Cavernous sinus thrombosis so there is no need for further tests. Start stat treatment with broad-spectrum IV antibiotics including antifungal and anticoagulation
  3. Carotid cavernous aneurysm so check intraocular pressure from right eye STAT
    4. Carotid cavernous fistula so order conventional digital subtraction angiography

  • The most common symptoms on carotid-cavernous fistula (CCF) are ocular bruit auscultated over the globe in high-flow CCFs, proptosis, chemosis, and conjunctival injection, ocular and/or orbital pain, headache, diplopia, and blurry vision.
  • Vascular etiologies can be seen on CTA, MRI, MRA, and orbital or transcranial ultrasound but conventional digital subtraction angiography is the gold-standard test for diagnosis of CCF.
  • CCFs may be an indirect (low-flow), or a direct (high-flow) CCFs based on a connection between the cavernous sinus and the intracavernous carotid artery versus the internal or external carotid artery, respectively.
  • The management of CCFs depends on the classification of CCFs, the onset of symptoms and the risk of long-term neurological impairment. Twenty to sixty percent of indirect CCFs will close spontaneously. Direct CCFs should be closed by transarterial/transvenous embolization or other treatment options if they are symptomatic and at risk of progression with attendant morbidity.
How well did you know this?
1
Not at all
2
3
4
5
Perfectly
2
Q

A 22-year-old male presents after a construction accident. Emergency medical services (EMS) reports there was a crane malfunction, and the patient was hit with a metal bar. Upon arrival, the patient has a Glasgow coma scale of 6, an initial pulse of 102 beats per minute, a blood pressure of 84/48 mm Hg, and an oxygen saturation of 92%. The patient is intubated for airway protection with cervical stabilization. The rest of the primary survey shows no abnormalities. Extended Focused Assessment With Sonography for Trauma. (EFAST) is negative. What is the next most appropriate step in management prior to computed tomography (CT) imaging?

A
  1. High dose methylprednisolone
  2. Phenylephrine
  3. Corticosteroids
    4. Intravenous fluids and packed red blood cell transfusion

  • This patient is an unstable trauma patient. Hypotension and tachycardia are classically seen in hemorrhagic shock. Hypotension and bradycardia are more characteristic of neurogenic shock and even in the setting of hypotension and bradycardia, one should first exclude and treat possible underlying hemorrhagic shock.
  • In accordance with ATLS, an undifferentiated trauma patient should be treated as having hemorrhagic shock until proven otherwise.
  • The initial treatment for hemorrhagic shock is volume repletion
  • If the patient is found to have spinal cord injury causing neurogenic shock and is appropriately fluid resuscitated, other treatments include vasopressors, anticholinergic medications such as atropine, and pure chronotropes such as isoproterenol. Consider the impact of other injuries when treating these patients.
How well did you know this?
1
Not at all
2
3
4
5
Perfectly
3
Q

A 30-year-old male patient presents with complaints of unremitting headaches, dysphagia, shoulder pain, neck pain, stiffness, and weakness. He complains that the pains keep him up at night, and he occasionally has night sweats. On physical examination, the patient appears cachectic, with a BMI of 17.9. On auscultation of the carotid arteries, no bruits are appreciated. On imaging, a mass appears to be occluding an area on the base of the skull lateral to the foramen magnum. What is the likely location of this patient’s lesion?

A
  1. Carotid canal
    2. Jugular foramen
  2. Internal auditory meatus
  3. Hypoglossal canal

  • The jugular foramen is located lateral to the foramen magnum
  • The jugular foramen carries the internal jugular vein as well as cranial nerves 9, 10, and 11. Compression of the foramen can cause damage or interruption of proper nerve function.
  • Spinal accessory nerve damage can result in shoulder pain, neck pain, shoulder weakness, neck weakness, and stiffness to these areas.
  • Damage to the vagus nerve and glossopharyngeal nerve can cause dysphagia, due to interruption of proper signal transmission via the nerves to its target head and neck muscles.
How well did you know this?
1
Not at all
2
3
4
5
Perfectly
4
Q

A 65-year-old male patient was diagnosed with tubercular spine disease and associated Frankel type D neurologic deficit six weeks ago. He was started on first-line anti-tuberculosis treatment (ATT). His image-guided biopsy revealed the growth of tubercle bacilli and good sensitivity to first-line ATT. His ESR and CRP have demonstrated an overall downward trend, and his back pain, as well as neurological status, have considerably improved. However, his MRI has shown a significant increase in the size of his abscess since the inception of chemotherapy. What is the appropriate next line of management?

A
  1. Repeat MRI in 6 weeks to assess the need for surgical intervention
    2. Continue antitubercular medications; repeat ESR, CRP, liver function tests (LFTs), and MRI at 6 weeks
  2. Consider the bacilli as drug-resistant and start second-line ATT 4. Abandon the non-operative line of management and immediately book the patient for surgery

  • Plain radiographs have an overall 15% sensitivity. In the early stages, with less than 30% vertebral destruction, they do not play much of a role. In later stages, with more than 30% vertebral destruction, patients can present with disc space reduction, endplate rarefaction, vertebral body destruction, instability, and spinal deformity. Computed tomography has almost 100% sensitivity. It can help in the diagnosis at a much earlier stage than plain x-rays. The types of vertebral destructive lesions based on CT in spinal TB include fragmentary, osteolytic, subperiosteal, and localized sclerosis. CT scan can also aid in image-guided biopsy for establishing the diagnosis.
  • Magnetic resonance imaging has almost 100% sensitivity and 80% specificity. MRI is the most useful modality in the diagnosis of spinal TB. The extent of soft tissue enhancement, the location of the abscess, and spinal canal compromise is best detected by MRI. Gadolinium-enhanced MRI may provide additional information regarding the diagnosis. Screening sequences involving the whole spine can also help in identifying non- contiguous vertebral involvement. MRI can also help in the assessment of response to treatment; however, immediately after starting antitubercular therapy, the MRI scan can reveal a worsening picture; and often lags behind the clinical improvement.
  • At the time of initial evaluation, patients are classified based on clinical examination and detailed radiological imaging (plain radiographs, CT, and MRI) into 5 stages: 1 - predestructive stage - hyperemia on scintiscan; 2 - early destructive stage - disc space reduction and paradisical erosion, knuckle less than 10 degrees, MRI demonstrates marrow edema, and CT shows erosions or cavitations; 3 - mild angular kyphosis, 2-3 vertebrae involved and kyphosis 10 - 30 degrees; 4 - moderate angular kyphosis, 2-3 vertebrae involved and kyphosis 30 - 60 degrees; 5 - severe angular kyphosis, >3 vertebrae involved and kyphosis greater than 60 degrees.
  • The most appropriate investigations at the time of follow-up in patients on treatment for TB include thorough clinical evaluation, blood inflammatory parameters (ESR, CRP), plain radiographs, and liver function tests to evaluate drug-associated complications. In a patient with evident clinical improvement and no specific concerns regarding these other targets at follow-up, no further change in antitubercular therapy or need for surgery is indicated. However, if a patient does not show any improvement clinically or based on inflammatory markers, or if they continue to worsen, all possibilities, including wrong diagnosis, poor immune status and nutrition, the inadequate dosage of drugs, drug resistance, or need for surgical intervention (possibility of underlying abscess which needs drainage) need to be considered. A further line of management needs to be individualized under such circumstances.
How well did you know this?
1
Not at all
2
3
4
5
Perfectly
5
Q

A 65-year-old man presents with complaints of painless visual blurring, which has happened twice in the last day. He has a history of hypertension, hyperlipidemia, and type 2 diabetes mellitus. He has a 30 pack-year smoking history, drinks alcohol occasionally, and has never used illicit drugs. He mentions that one day earlier, he lost his speech for a few minutes. On examination, it is determined that he has again lost vision in his left eye. What is the most appropriate next step in the management of this patient?

A
  1. Heparin infusion
  2. Fibrinolytic therapy
    3. CT brain scan without contrast
  3. Acetylsalicylic acid

  • Before beginning lytic therapy for a developing stroke, it is most essential to obtain CT brain imaging to ensure that there is no hemorrhagic component.
  • Only embolic and ischemic strokes can be treated with lytic therapy. It is very dangerous to treat hemorrhagic strokes with lytic therapy.
  • In this clinical scenario, painless loss of vision may be amaurosis fugax and may be associated with carotid artery stenosis.
  • Thus, another recommended study is a duplex ultrasound of the neck to look at the carotid bifurcation. Alternatively, one could follow the unenhanced brain CT with a head and neck CT angiogram.
How well did you know this?
1
Not at all
2
3
4
5
Perfectly
6
Q

An 87-year old female with a history of osteoporosis sustains a ground level fall. She complains only of lower back pain. She is taken to the local hospital where a physical examination reveals only focal tenderness to the lumbar spine and no evidence of neurological deficits. Advanced imaging reveals a spinal fracture of L2 that involves only the anterior 1/2 of the vertebral body. What type of fracture is this according to the Denis classification?

A

1. Compression
2. Burst
3. Flexion-distraction
4. Fracture-dislocation

  • Compression fractures, as defined by Denis, involve injury to the anterior column of the vertebral body with an intact middle column.
  • These are inherently stable fractures.
  • A burst fracture is defined as fractures of the anterior and middle columns with possible additional involvement of the posterior column if lamina fractures are present or if dural tears or nerve entrapments occur. In this question, only the anterior column is injured.
  • A flexion-distraction injury describes an injury incurred on all three columns. In this question, only the anterior column is affected. A fracture-dislocation injury describes an injury incurred on all three columns, with translation. In this question, only the anterior column is affected.
How well did you know this?
1
Not at all
2
3
4
5
Perfectly
7
Q

A 65-year-old female was brought to the neurology clinic following sudden onset dropping of her bilateral eyelids. She has no such previous episodes. On examination, she was conscious, oriented, and had no weakness or ophthalmoplegia. However, her left pupil was dilated, and the horizontal pursuit and saccadic eye movements were affected. An urgent MRI brain was advised. Based on the clinical scenario, MRI is most likely to reveal stroke at which region?

A
  1. Anterior midbrain
    2. Posterior midbrain
  2. Anterior pons
  3. Posterior pons

  • The involvement of the single caudally placed levator palpebrae superioris nucleus that innervates both the eyelid elevators can give rise to bilateral ptosis due to stroke within the oculomotor nuclear complex.
  • The neural pathway governing the horizontal pursuit and saccadic eye movements also passes through the tegmentum near the oculomotor nucleus.
  • The involvement of the rostral Edinger-Westphal nuclear component within the oculomotor nuclear complex gives rise to an ipsilateral dilated pupil.
  • A stroke affecting the posterior midbrain region and affecting the oculomotor nuclear region would account for the pattern of clinical deficits seen in the patient.
How well did you know this?
1
Not at all
2
3
4
5
Perfectly
8
Q

A patient has edema and a dilated cardiomyopathy due to a dietary deficiency. What is the most likely etiology?

A

1. Thiamine deficiency
2. Vitamin C deficiency
3. Niacin deficiency
4. Pyridoxine deficiency

  • This patient has wet-beriberi, which results from thiamine (B1) deficiency.
  • In the developed world, thiamine deficiency most commonly occurs in patients with alcohol use disorder.
  • Thiamine deficiency also causes Wernicke-Korsakoff and dry beriberi.
  • Dry beriberi manifests as polyneuritis and symmetrical muscle wasting.
How well did you know this?
1
Not at all
2
3
4
5
Perfectly
9
Q

A 36-years-old female presents to the hospital for back pain and fever. Symptoms started four days ago and are getting progressively worse. She was seen in an urgent care center and was given pain medication and antibiotics with no improvement. She had a skin infection some three weeks ago that lasted a few days but disappeared without treatment. Otherwise, she is healthy and takes no medication. Physical exam shows enderness over the thoracic spine, numbness in both thighs and low-grade fever. Her blood count shows leukocytosis. Blood cultures are drawn. The emergency department does not have the capability for emergency MRI. Which of the following is the next best step in the management of this patient?

A
  1. MRI of the spine with contrast to be done outpatient for better sensitivity and wait for results of culture to be available
    2. CT of the spine with contrast followed by neurosurgical evaluation; start antibiotics (vancomycin and cefotaxime)
  2. CT scan of the spine without contrast followed neurosurgical evaluation; start antibiotics (vancomycin and ceftriaxone)
  3. CT scan of the spine with contrast followed by neurosurgical evaluation; start antibiotics (vancomycin and piperacillin/tazobactam)

  • Spinal epidural abscess (SEA) is a true medical emergency. 1/3 of patients have no clear etiology for acquiring the infection. Of the other 2/3, the more common site of origin is skin infection followed by complications of spinal surgery.
  • Staphylococcus is the organism in the majority of cases followed by gram-negative bacilli.
  • CT and MRI with contrast are needed for SEA diagnosis. MRI is preferred, but if not immediately available, CT is a feasible alternative.
  • Vancomycin plus a later generation cephalosporin (cefotaxime, ceftriaxone, cefepime) is the empiric antibiotic treatment of choice. If Pseudomonas is suspected (prior surgery or invasive procedures) then ceftazidime is preferred.
How well did you know this?
1
Not at all
2
3
4
5
Perfectly
10
Q

A 16-year-old girl who has undergone ventriculoperitoneal shunting for aqueductal stenosis 12 years ago was brought in with a history of high-grade fever, persistent vomiting, and altered sensorium. The child was drowsy with a withdrawal motor response to a painful stimulus. She had neck rigidity as well. A shunt infection was diagnosed. What would be the most rational management strategy in this case?

A
  1. Remove the shunt, place a new shunt, and start high-dose antibiotics
  2. Exteriorize the proximal shunt as external ventricular drainage (EVD), start antibiotics and then place a new shunt once the infection is controlled
  3. High-dose antibiotics alone
    4. Exteriorize the proximal shunt as EVD, start antibiotics and then plan for endoscopic third ventriculostomy (ETV); once the infection is controlled, and the CSF culture is sterile

  • In cases of shunt infection, the ideal approach is to exteriorize its proximal part as EVD and then start antibiotics as per the CSF culture and sensitivity reports.
  • Once the culture is sterile, to minimize the risk of subsequent shunt infection, it is a foreign body and a probable site of bacterial colonization; ETV can be undertaken in this patient since it is of an obstructive pattern of hydrocephalus.
  • ETV has a high success rate in managing such an obstructive type of hydrocephalus without concurrent risk of infection and obstruction of the shunt tubes.
  • Only antibiotics would have less chance of recovery in shunt infection. Placing a new shunt after controlling infection is an option in cases of communicating patterns of hydrocephalus.
How well did you know this?
1
Not at all
2
3
4
5
Perfectly
11
Q

A 47-year-old female presents to a pain clinic following the acute onset of severe neck pain radiating to her left arm for the last four days. MRI spine revealed severe left foraminal stenosis following a disc prolapse at the C5-C6 level. The pain has been refractory to pain medications. The treating clinician plans cervical traction therapy for the patient. The patient has a history of heavy smoking and is on bronchodilators for chronic obstructive pulmonary disease. There is no history suggestive of exertional dyspnea or orthopnoea. Which of the following modes of traction is most appropriate for this patient?

A
  1. Supine with the neck in the neutral position
    2. Supine with neck flexion at 20 to 30 degrees
  2. Supine with neck extension at 20 to 30 degrees
  3. Traction in a sitting position

  • The patient has clinical signs, and symptoms that suggest cervical radiculopathy following acute disc prolapse.
  • The cervical traction is more effective in a supine position, with the neck flexed to 20 to 30 degrees.
  • This allows the most effective separation of the intervertebral disc spaces at C4-7 following elongation of the posterior muscular components aided with a pull from a correct load.
  • Although the patient is known to have chronic obstructive pulmonary disease, her respiratory functional status is stable. She could handle being in the supine position, which would yield more effective traction than in the sitting position (allowing for more posterior pressure loading). The sitting position is only used in patients strictly cannot lie on their backs (e.g., people who suffer from reflux esophagitis).
How well did you know this?
1
Not at all
2
3
4
5
Perfectly
12
Q

A 3-month-old male arrives with a sacral dimple in the midline and peripheral darkened, coarse black hair surrounding the dimple. The only notable finding on the exam is that the child’s left calf is approximately 6 millimeters smaller than his right calf. Which of the following is the next best step in the management of this patient?

A
  1. Ultrasound of the lumbosacral spine
    2. MRI imaging of the lumbosacral spine
  2. Plain lateral and AP films of the lumbosacral spine
  3. No further workup is warranted

  • Even though an ultrasound of the lumbosacral spine could delineate a low lying conus, it would be difficult to diagnose a fatty filum using ultrasound alone. In addition there is a high rate of false-positives for a low lying conus. MRI imaging of the lumbosacral spine would be the next best step in evaluating this child.
  • Definitive imaging using MRI is required to evaluate the patient for significant findings including fatty filum/tethered cord, syrinx, and/or Chiari I malformation. Defining the underlying anatomy would be the next appropriate step to allow the clinician to make appropriate treatment decisions.
  • Plain lateral and AP films of the lumbosacral spine would not be useful in further evaluating this child. Any finding of bony abnormalities which still require subsequent MRI imaging to determine the necessity of surgical intervention. Additionally, plain images would not allow for evaluation of a low lying conus or fatty filum. Additionally, a syrinx would not be visualized on these studies.
  • No further workup would not be appropriate given the presence of a sacral dimple, associated hairy patch, and muscle asymmetry of the lower extremities. Definitive imaging using MRI would be required to evaluate the patient for important findings including fatty filum/tethered cord, syrinx, and/or Chiari I malformation. Not defining the underlying anatomy would preclude the clinician from making the appropriate treatment decisions.
How well did you know this?
1
Not at all
2
3
4
5
Perfectly
13
Q

A 58-year-old male presents with complaints of homonymous hemianopsia of his left visual field and headaches for the past 2 months. MRI of the brain shows a lesion of the right occipital lobe, and glioma is suspected. Surgery for the resection of the tumor in the occipital lobe is planned. Which of the following is an important anatomical reference of approach from the medial surface of the occipital lobe?

A
  1. Transverse sulcus
    2. Calcarine sulcus
  2. Lingual gyrus
  3. Lunate sulcus

  • The calcarine sulcus divides the medial surface into the cuneus gyrus and lingual gyrus. The transverse sulcus is present on the lateral surface of the occipital lobe.
  • The calcarine sulcus is invariably identified in the posterior interhemispheric fissure region. It has a minimal number of side branches and always arises from the parahippocampal gyrus. Thus, making it an important landmark for surgical approaches of the area.
  • The calcarine sulcus extends from the occipital pole to the parieto-occipital sulcus. The lingual gyrus is present on the lateral surface of the occipital lobe.
  • The calcarine sulcus divides the medial surface into the cuneus gyrus and lingual gyrus. The primary visual cortex lies in the upper and lower banks of the calcarine sulcus and can manifest with contralateral homonymous hemianopsia when compromised.
How well did you know this?
1
Not at all
2
3
4
5
Perfectly
14
Q

A 55-year-old woman is seen after a basilar artery stroke. She has had symptomatic carotid artery stenosis for the past year and has refused to see a surgeon for correction during that time. Imaging shows a persistent, right-sided primitive artery branching proximally to the carotid canal and entering the cranium via an opening directly posterior to the carotid canal. As it enters the cranium, it terminates at the basilar artery, forming an anastomosis. This is the suspected cause of the stroke in this patient, and a surgeon plans for correction. The procedural plan is to approach through a major vessel and tie off the anomalous artery just before it enters the cranium. The surgeon makes note of the nearby structures and proceeds. The procedure is successful; however, on postoperative examination, the patient has developed dysphagia and notes numbness in her throat. Which foramen was accessed during the procedure, and which nerve was most likely injured?

A
  1. Foramen lacerum: nerve of pterygoid canal
  2. Foramen spinosum: nervus spinosus
  3. Supraorbital foramen: trigeminal nerve
    4. Jugular foramen: glossopharyngeal nerve

  • The carotid canal lies immediately anterior to a deep depression, the jugular foramen, formed by the petrous portion of the temporal bone and jugular process of the occipital bone.
  • The carotid artery can rarely have a persistent artery that uses the jugular foramen to enter the cranium, which can cause carotid artery stenosis.
  • The jugular foramen contains the glossopharyngeal nerve. Injury to this area can result in glossopharyngeal nerve palsy (diminished sensation posterior tongue, palate, and pharynx as well as dysphagia).
  • The basilar artery is located on the anterior portion of the brain stem and enters the cranium through the foramen magnum. The foramen magnum is closely medial to the jugular foramina.
How well did you know this?
1
Not at all
2
3
4
5
Perfectly
15
Q

A 28-year-old woman with a past medical history of Chiari malformation undergoes posterior fossa decompression in the sitting position. After the procedure, she develops new-onset left- sided facial droop and right-sided weakness. What is the most appropriate treatment for this patient’s condition?

A
  1. Tissue plasminogen activator (TPA)
  2. Craniotomy
  3. Bilevel positive airway pressure (BiPAP)
    4. Hyperbaric oxygen therapy (HBOT)

  • Neurosurgical procedures in the sitting position have a high risk for air embolism.
  • The most appropriate treatment for this patient is hyperbaric oxygen therapy (HBOT)
  • HBOT helps to reduce the size of air emboli.
  • It also can facilitate the diffusion of nitrogen from the emboli back into the bloodstream.
How well did you know this?
1
Not at all
2
3
4
5
Perfectly
16
Q

A 67-year-old African American male with metastatic rectal cancer was referred to the interventional pain physician by the oncologist because of intractable back pain. His pain is 10/10 on a high dose of fentanyl patch and oral extended release morphine. Subarachnoid neurolysis was planned. Which of the following is correct?

A
  1. He would benefit from 4mg IV midazolam to help with anxiety during the procedure.
  2. He will need admission overnight to monitor for any hypotension from the subarachnoid block
    3. He will need to lay still for at least half an hour after the procedure is done
  3. This procedure would give him 100 percent pain relief for the next 6 months

  • This procedure requires continuous communication with the patient to ensure the needle is in the right level. The provider injects a small quantity (0.1 ml) of alcohol and confirms with the patient that pain in the targeted level. Therefore, any sedative- hypnotic agent should be avoided.
  • The concentration of phenol and alcohol in cerebrospinal fluid diminishes rapidly after administration. The patient should be maintained in a position that is often uncomfortable for at least 30 minutes after the injection. It is important to prevent leakage of the alcohol or phenol to unintended nerve roots. This is why the patient should be maintained in a position.
  • While the neuraxial neurolysis is not always permanent, there is unlikely to be 100% relief in a patient with metastatic cancer.
  • Up to 60% of patients have good pain relief after neuraxial neurolysis.
How well did you know this?
1
Not at all
2
3
4
5
Perfectly
17
Q

A 42-year-old female presents with low intermittent back pain for 6 months which worsens as she bends to pull weeds in her garden. The pain improves when she reclines in her recliner. A plain film x-ray of the lumbar spine is reported as showing “mild grade I spondylolisthesis without significant change between flexion or extension and without other significant abnormalities.” What is the probability that she will fail conservative therapy and require surgical treatment?

A

1. 10% to 15%
2. 45% to 55%
3. 75% to 85%
4. 85% to 95%

  • Conservative therapy will fail in about 10% to 15% of younger individuals with low-grade nontraumatic spondylolisthesis.
  • As one ages, conservative treatment can help with low-grade slippage.
  • The role of surgery is to decompress the nerves and immobilize the unstable segment(s). Fusion is often necessary at the level of the intervertebral discs.
  • Short-term results reveal that surgery has markedly better results compared to conservative approaches, especially in young people.
How well did you know this?
1
Not at all
2
3
4
5
Perfectly
18
Q

A 44-year-old male presents with complaints of weakness in both hands that have gradually developed over the last three months. Additionally, he has mild swallowing difficulty and occasional speech slurring. On physical examination, there is marked atrophy of intrinsic hand muscles with diffusely hyperactive deep tendon reflexes and positive Babinski reflex. An electromyogram (EMG) and nerve conduction study (NCS) are ordered. These tests will most likely show which of the following?

A
  1. Slow nerve conduction velocities
  2. Decremental response on repetitive stimulation of motor nerves
  3. Absent sensory nerve responses
    4. Fibrillations on needle EMG

  • The patient likely has amyotrophic lateral sclerosis, demonstrated by both upper and lower motor neuron findings on exam.
  • EMG and NCS exams most commonly show evidence of active denervation on needle EMG (fibrillations and positive sharp waves) with possible reduced amplitudes of motor nerve conductions due to loss of viable axons supplying the muscles.
  • Sensory nerve conductions would be preserved throughout the disease, and motor nerve velocities would be unaffected until very late in the ALS disease process.
  • Unfortunately, the diagnosis of ALS is not easy, and it can take months to work up the patient since there is no specific test. Nerve conduction studies and needle EMG can help confirm the diagnosis and exclude other peripheral neurological disorders like demyelinating neuropathies and neuromuscular transmission defects like myasthenia gravis (decremental response seen on repetitive motor nerve stimulation).
How well did you know this?
1
Not at all
2
3
4
5
Perfectly
19
Q

A 24-year-old professional gymnast with no past medical history presents with lower back pain that has been present for 3 months. The patient states the pain is exacerbated when he extends his spine. Upon physical examination, placing the patient into lumbar extension exacerbates his pain. Lateral lumbar x-ray shows anterior displacement of L4 in correlation to L5. What is the most common cause of this finding?

A
  1. Spondylolisthesis
    2. Spondylolysis
  2. Anterolisthesis
  3. Osteoarthritis

  • Spondylolysis is a fracture of the pars interarticularis; this is also known as a “Scottie Dog fracture” which can be missed on the lateral lumbar x-ray, and more commonly seen on the oblique lumbar x-ray.
  • Spondylolysis is the most common cause of spondylolisthesis. Spondylolysis occurs in patients who may overwork themselves and participate in high impact activities that involve extension and rotation, which is seen in gymnasts.
  • Spondylolysis is seen in the L5 region in approximately 90% of cases.
  • The patient has spondylolisthesis, specifically anterolisthesis of L4 on L5, meaning there is anterior slippage of the vertebral body in correlation to the one below it. The most common cause of spondylolisthesis is spondylolysis. Anterolisthesis is a specific type of spondylolisthesis; it is specifically anterior displacement in correlation to the one below it. The patient in this scenario does have symptoms and radiographic imaging consistent with anterolisthesis, but the question is specifically asking what the most common cause of anterolisthesis is, for which the answer is spondylolysis. Osteoarthritis is a degenerative joint disease that is due to wear and tear to the articular cartilage. Although this may be a contributing factor to the patient’s pain, this patient does not fall within the common image of an osteoarthritis patient. Osteoarthritis patient risk factors are obesity, age, and females. This patient is a healthy professional gymnast who fits the criteria more for spondylolysis, which is high impact, overuse, extension, and rotation exercises. Radiographic imaging for patients with osteoarthritis will show spondylosis, which is degenerative changes in the discs, vertebral bodies, and z joints. Spondylolysis, on the other hand, is a fracture of the pars interarticularis.
How well did you know this?
1
Not at all
2
3
4
5
Perfectly
20
Q

A 25-year-old male suffers from a transient inability to perform calculations and write. After resolution, he has no residual problems. He has no other neurological disorders. Which of the following tests should be performed in this patient?

A
  1. CT scan without contrast
    2. EEG
  2. CT scan with contrast
  3. MRI

  • This patient is most likely suffering from ictal Gerstmann syndrome. An EEG should be performed to confirm the diagnosis.
  • CT scan without contrast can be done in this patient to exclude a hemorrhage as a cause of his epileptic syndrome, but EEG is superior in confirming the ictal form of this syndrome.
  • CT scan with contrast can be done in this patient to exclude tumors as a cause of his epileptic syndrome, but EEG is superior in confirming the ictal form of this syndrome.
  • MRI can be done in this patient to exclude tumors as a cause of his epileptic syndrome, but EEG is superior in confirming the ictal form of this syndrome.
How well did you know this?
1
Not at all
2
3
4
5
Perfectly
21
Q

A 16-year-old girl is involved in a serious motorcycle accident. The patient exhibits a fracture-dislocation of a vertebra. The signs and symptoms include (1) bilateral loss of pain and thermal sense within the area of the fracture, (2) lower motor neuron symptoms and vasomotor paralysis in areas supplied by the injured segments on the right side, (3) loss of sensory impulses in the posterior white columns below the lesion on the right side, and (4) upper motor neuron lesion below the level of injury on the right side. The symptoms described are most suggestive of which of the following?

A
  1. Traumatic syringomyelia
  2. Anterior spinal cord syndrome
  3. Posterior spinal cord syndrome
    4. Brown-Sequard syndrome

  • Brown-Sequard syndrome characteristically results following hemisection of the spinal cord.
  • It is characterized by loss of motor function on the side of the lesion and loss of protopathic sense on the contralateral side.
  • It is most commonly caused by penetrating or blunt trauma but can also be caused by mass occupying lesions, bleeding disorders, and disc herniations.
  • Syringomyelia typically results in the loss of pain and temperature sensations in the cape-like patterns. Anterior cord syndrome results in loss of motor as well as pain and temperature below the level of spinal injury. Posterior cord syndrome results in loss of proprioception and vibrations sensation below the level of injury
How well did you know this?
1
Not at all
2
3
4
5
Perfectly
22
Q

A 28-year-old G2P1 woman notes that fetal movements were less during this pregnancy than in prior pregnancies. At birth, the infant is floppy and has very feeble arm and leg movements. The infant cannot feed, has difficulty breathing, and has a weak cry. What is the most likely diagnosis?

A
  1. Cerebral palsy
  2. Duchenne muscular dystrophy
    3. Werdnig-Hoffmann disease
  3. Polio

  • Werdnig-Hoffmann disease, or type 1 spinal muscular atrophy, is characterized by a floppy baby. Marked hypotonia at birth is present.
  • Weakness is a prominent feature, and proximal weakness predominates.
  • Affected children can never sit or stand up and may require prolonged respiratory support to survive.
  • Children born with Werdnig-Hoffman disease have normal intelligence.
How well did you know this?
1
Not at all
2
3
4
5
Perfectly
23
Q

A 35-year-old male patient presented with chronic, daily headaches localized to the occipital area and radiating to shoulders bilaterally. Symptoms are not responsive to analgesic medications. He reports occasional gait instability and episodes of dizziness. MRI of the brain and spine showed Chiari malformation type 2. The patient undergoes foramen magnum decompression. At the four-week follow-up visit, the patient reports left-sided shoulder weakness with dropping of the left shoulder and winging of the scapula. There is also some loss of shoulder abduction. What could be the cause of this impairment?

A
  1. Prolonged anesthesia affecting plexus
  2. Muscle dissection leading to nerve injury
  3. Bone fracture due to abnormal position during surgery
    4. Failure of postoperative physical therapy

  • Spinal nerve injury could occur during the posterior fossa decompression surgery for Chiari malformation.
  • The surgical position such as semi-sitting could be the main cause of stretching the nerve due to the weight from the unsupported arms. Moreover, a prone position could stretch the nerve when too much retraction is used to hold the shoulders.
  • Another possible source of injury to the accessory nerve is a thermal injury during muscle dissection along or near the course of the nerve.
  • Using self-retainers and having a thin neck (with small muscle bulk) could lead to excessive stretching of the accessory nerve, causing nerve injury.
How well did you know this?
1
Not at all
2
3
4
5
Perfectly
24
Q

A 25-year-old woman admitted to the hospital suffering from pain in the pectoral and axillary region, weakness and muscle loss at the base of the thumb. An x-ray result shows that the patient has a cervical rib. What is the most likely cause of these symptoms?

A
  1. Arterial thoracic outlet syndrome
  2. Venous thoracic outlet syndrome
  3. Rib tumor
    4. Neurogenic thoracic outlet syndrome

  • The cervical rib commonly causes the neurogenic thoracic outlet syndrome by compressing the lower trunk of the brachial plexus.
  • The base of the thumb muscle’s function is lost due to the compression of the lower trunk; because the lower trunk is the origin of the nerves innervating this muscle.
  • The syndrome is common in females.
  • The onset of the syndrome is common in people who are 20-50 years of age.
How well did you know this?
1
Not at all
2
3
4
5
Perfectly
25
Q

A 55-year-old male with altered cognition and right-sided weakness was diagnosed with primary angiitis of the central nervous system (PACNS) following cerebral angiography and stereotactic biopsy. He was promptly started on combination therapy with oral prednisone and cyclophosphamide. During the follow-up visit, the patient had no resolution of prior symptoms and was noted to have developed new-onset blurring of vision, which was also determined to be secondary to PACNS. Which of the following is the next best step in the management of this patient?

A
  1. Increase the dose of cyclophosphamide and prednisone 2. Discontinue both the drugs and start on intravenous methylprednisolone
  2. Discontinue both drugs and start infliximab
    4. Add infliximab to the current drug regimen

  • The patient has features consistent with primary CNS vasculitis refractory to both glucocorticoids and immunosuppressants, and therefore adjunctive therapy is advocated.
  • Tumor necrosis factor-alpha blockers (infliximab, etanercept) and mycophenolate mofetil can be used as adjunctive therapy along with the standard therapy.
  • Patients started on tumor necrosis factor-alpha blockers have shown striking resolution of their neurological as well as radiological characteristics.
  • The evidence of intravenous (IV) steroids being superior to oral regimens has not been noted. The discontinuation of the primary therapy is not advocated in refractory cases of CNS vasculitis.
How well did you know this?
1
Not at all
2
3
4
5
Perfectly
26
Q

A 45-year-old male from Colombia has a brain magnetic resonance image (MRI) performed for evaluation of chronic headaches with findings of multiple brain lesions. The lesions show are consistent with vascular lesions on susceptibility-weighted imaging (SWI) with a “popcorn” like appearance. Which of the following advice should be given to his son and daughter?

A

1. Genetic counseling should be offered to both son and daughter
2. The son and daughter should be referred for a catheter angiogram to evaluate for vascular malformations
3. Genetic counseling should be offered to the daughter but does not need to be offered to the son
4. No specific advice is required as the cavernous malformations likely occurred de novo

  • Cavernous malformations can be either sporadic or familial in etiology.
  • Between 40% to 60% of cases of cavernous malformations are familial.
  • Familial cases of cavernous malformations are inherited in an autosomal dominant fashion. First-degree relatives of patients with familial type cavernomas should have a screening MRI and genetic counseling.
  • Familial cases of cavernous malformations tend to present with multiple cavernomas, compared with solitary cavernomas in sporadic occurrences. A higher proportion of Hispanic patients with cavernomas have familial type cavernomas compared with other populations.
How well did you know this?
1
Not at all
2
3
4
5
Perfectly
27
Q

A 35-year-old man presents to the clinic for follow-up. He has presented to the emergency department twice over the past two months with recurrent episodes of left hemiparesis and left homonymous hemianopia which resolved spontaneously. His head CT and subsequent MRI showed a right deep hemisphere infarction. Subsequent CT angiogram showed complete occlusion of his right terminal internal carotid artery with a mesh of distal collaterals. His right internal carotid and middle cerebral arteries were normal. The patient has no history of hypertension, diabetes, or hyperlipidemia. He does not smoke. He does have a history of craniopharyngioma, which was resected when he was a child with good recovery. What is the next best test to determine the efficacy of treatment for him to prevent subsequent ischemic events?

A
  1. Cardiac echocardiogram
  2. CT angiogram of his neck arteries
  3. Platelet function tests
    4. Acetazolamide challenge with CT angiogram

  • The patient has had recurrent ischemic events related to underlying right Moyamoya disease.
  • The most definitive therapy is revascularization with direct extracranial to intracranial bypass grafting or indirect revascularization with omental or dural synangiosis.
  • Angiography with acetazolamide and carbon dioxide reactivity are the best tests to determine vascular reactivity and subsequent response to revascularization.
  • The patient most likely has terminal right ICA occlusion with Moyamoya collaterals. A neck angiogram will not be helpful at all. Cardioembolism is one of the main causes of stroke in young patients, yet the description of this patient is clearly not embolic. Antiplatelet agents are indicated in secondary prevention of stroke in this patient, yet there is no evidence platelet function tests will help to determine the efficacy of its treatment.
How well did you know this?
1
Not at all
2
3
4
5
Perfectly
28
Q

A 72 -year-old male underwent coronary artery bypass grafting. Post-operatively he is diagnosed with ventral cord syndrome at the T6-T7 level on an MRI. What findings will a neurological exam likely show?

A
  1. A motor loss in the upper extremities
  2. Hyperactive reflexes
  3. Loss of proprioception
    4. Flaccidity with absent reflexes

  • T6-T7 is below the innervation of the upper extremities, so these would not be affected.
  • Acute VCS will have absent reflexes. However, there is a gradual return of the reflexes with increased tone or spasticity later on.
  • Position sense is spared throughout the spinal cord.
  • The acute stages of VCS present flaccidity and loss of deep tendon reflexes. Further, spasticity and hyperreflexia will develop in the following days and weeks.
How well did you know this?
1
Not at all
2
3
4
5
Perfectly
29
Q

Midway through surgery, a patient’s somatosensory evoked potentials (SEPs) have been stable and consistent. However, over the course of a few minutes, the amplitude drops to 40% of baseline, and the latency increases by 15%. It is thought to be most likely due to drops in the patient’s temperature. Which measure would be most effective in reversing the observed SEP changes?

A
  1. Increase the forced air warmer temperature from 40 C to 43 C
    2. Ask the surgeon to flood the surgical field with warmed saline 3. Increase the temperature of the room by 3 to 5 degrees
  2. Apply several warm blankets to exposed areas of the patient

  • Many surgical factors can alter SEPs, though the most time- sensitive of these are ischemia, surgical manipulation of structures, cautery, and dissection. Ischemia could be caused by pressure, retraction, clipped vessels, etc.
  • Manipulating structures and dissecting in certain areas may compromise neurologic structures and/or tracts, leading to a loss or distortion of evoked potential signals that, if not corrected, could result in permanent neurologic deficits. Cautery, if used too liberally, could compromise blood flow and/or directly injure a nerve or spinal tract.
  • Flooding the surgical field with warmed saline would most quickly and effectively warm the components of the sensory pathway being monitored with SEPs, which have likely been exposed to room air and become relatively hypothermic.
  • Although this would likely increase the patient’s body temperature given an adequate amount of time, it is not the fastest way to address the SEP changes, nor is it as effective as applying a warm solution to the surgical field, where the sensory path is likely exposed and vulnerable to being cooled to ambient temperatures. Attempts at directly warming the portions of the sensory pathway that are exposed during surgery, such as flooding the surgical field with warm saline, are likely to be more successful more quickly than attempting to warm only exposed parts of the patient.
How well did you know this?
1
Not at all
2
3
4
5
Perfectly
30
Q

A patient presents with sudden loss of vision in his right eye that lasted 45 seconds. He also felt weakness in his left arm at the same time, but the symptoms improved rapidly. He is on metoprolol, aspirin, and atorvastatin. Work up reveals a bruit in the neck and a duplex ultrasound reveals 90% stenosis at the right carotid artery bifurcation. However, because of ongoing chest pain, it is decided to stent the lesion. It is decided to perform the carotid artery stenting under intravenous sedation. After the introduction of the lead wire into the neck, an anti-embolic protection device is inserted before angioplasty and stent deployment. Once the stent is applied, the physician is unable to retrieve the anti-embolic device. What should be the next step?

A
  1. Make a neck incision
  2. Turn the head to the ipsilateral side
    3. Turn the head to the contralateral side
  3. If the carotid artery is patent, leave the device alone

  • As endovascular procedures grow in numbers, more complications are coming to light.
  • During carotid artery stenting, sometimes the anti-embolic device may get stuck, but the device may come loose by altering the head position.
  • However, there is a risk of a stroke when manipulating this device.
  • Sometimes one may have to gently extend the neck to dislodge the device. The aim is to straighten the internal carotid artery. If preoperatively the internal carotid artery is tortuous, expect this complication to occur.
How well did you know this?
1
Not at all
2
3
4
5
Perfectly
31
Q

A 35-year-old male patient is undergoing endoscopic thoracic sympathectomy for benign paraganglioma near T2 of his spine. Sectioning of the rami communicantes between the sympathetic trunk and spinal nerve of T2 would result in which of the following?

A
  1. Paralysis of all intercostal musculature below T2
  2. Lack of sensation in intercostal spaces 2 and 3
  3. Ptosis of the eyelid
    4. Lack of axillary sweating

  • In patients with a history of spinal cord injuries, autonomic dysreflexia can lead to increased axillary sweating. Resection of the rami communicantes, transmitting pre- and post-ganglionic sympathetic nerve fibers, would not affect the somatic motor fibers running in the spinal nerves and thus, there would be no paralysis of intercostal musculature.
  • Hyperhidrosis of the axillary sweat glands might be a result of overstimulation of the sympathetic nervous system via the rami communicantes, therefore a lack of sweating would result from an interruption of this system. Resection of the rami communicantes, transmitting pre- and post-ganglionic sympathetic (visceral efferent) nerve fibers, would not affect the somatic afferent fibers running in the spinal nerves. Thus, there would be no general sensory loss in the affected intercostal spaces or the subjacent parietal pleura.
  • The superior tarsal muscle receives innervation by sympathetic fibers exiting the spinal cord at T1, synapsing in the superior cervical ganglion, and eventually distributed to the orbit via the oculomotor nerve. The superior tarsal muscle serves to raise the superior eyelid. Its innervation would not be affected by a division of sympathetic innervation below T1, as in this case.
  • Sympathectomy is surgical resection of the sympathetic nervous innervation to reduce symptoms such as hyperhidrosis (excessive sweating). The thoracic sympathetic outflow to the axillary sweat glands would be disrupted by cutting the rami communicantes at the second thoracic spinal level.
How well did you know this?
1
Not at all
2
3
4
5
Perfectly
32
Q

A 75-year-old with a history of berry aneurysms presents to the ED because of the sudden onset of diplopia. On examination, there is a lack of ability of the right eye to adduct in the primary position (looking straight ahead). All other extraocular eye movements are normal. What would you expect to find if you performed a swinging flash light test on this patient?

A
  1. Shining light in right eye leads to constriction of the right pupil with consensual constriction of the left pupil
  2. Shining light in right eye leads to constriction of the right pupil without consensual constriction of the left pupil
    3. Shining light in left eye leads to constriction of the left pupil without consensual constriction of the right pupil
  3. Shining light in left eye leads to constriction of the left pupil with consensual constriction of the right pupil

  • Oculomotor nerve palsies are most commonly caused by microvascular complications of hypertension or diabetes, trauma, or compression via aneurysms or neoplasm.
  • The oculomotor nerve handles most movements of the eye, including adduction in primary position (looking straight ahead). In addition to somatic innervation, the oculomotor nerve controls parasympathetic innervation to the pupil and lens. These fibers are located on the outermost part of the nerve, and are thereby prone to compression from the outside via aneurysms, neoplasm, or increased intracranial pressure.
  • The pupillary reflex is controlled by the optic nerve (afferent) and oculomotor nerve (efferent). Due to the anatomy at the brainstem, when one optic nerve is stimulated both oculomotor nerves become stimulated. This results in bilateral pupil constriction when light is shined in one eye.
  • If the light is shined in one eye and neither pupil constricts, this is likely an optic nerve pathology because the stimulus did not reach the brainstem via the afferent pathway. If the light is shined in one eye and the same eye constricts, but there is no consensual constriction of the other eye, the oculomotor nerve on the other side is likely affected. If the light is shined in one eye and only the other eye constricts, the pathology probably lies within the oculomotor nerve on the same side.
How well did you know this?
1
Not at all
2
3
4
5
Perfectly
33
Q

A 72-year-old female with a known diagnosis of obesity hypoventilation syndrome presented to the sleep clinic with complaints of progressively worsening nocturnal choking episodes, despite using continuous positive airway pressure (CPAP). The PaCO2 and pulse oximetry levels obtained in the clinic were 40 mmHg and 80%, respectively. What should be the ideal therapeutic option for this patient at this time?

A
  1. Weight loss surgery
    2. Supplemental oxygen therapy
  2. Tracheostomy
  3. Medroxyprogesterone

  • According to current guidelines, if pulse oximetry levels remain persistently below 90% despite the CPAP therapy, supplemental oxygen therapy can be tried.
  • Weight loss surgery is usually the last resort.
  • Bi-PAP can also be tried in patients with increasing episodes of apnea or hypopnea on CPAP therapy.
  • Medroxyprogesterone use is not regularly recommended.
How well did you know this?
1
Not at all
2
3
4
5
Perfectly
34
Q

A 72-year-old white man with prostate cancer treated ten years ago with resection presents with the complaint of double vision and not being able to completely open the eyelid on the right side over the past two months. The patient denies any other focal weakness, sensory loss, difficulty breathing, eating, or coughing. Vitals are normal. The neurological exam showed restriction of right eye abduction and right side ptosis with miosis. The fundal exam is unremarkable. Miosis is not changed after cocaine eye drop to the right eye, but the left eye dilated. Basic serum blood workup and CT head are unremarkable. Which of the following structures are involved in having these symptoms?

A
  1. Trochlear nerve and ophthalmic nerve
    2. Abducens nerve and sympathetic fibers
  2. Oculomotor nerve and sympathetic fibers
  3. Maxillary Nerve and abducens nerve

  • If anisocoria increases and one eye shows minimal dilation with cocaine eye drop, it indicates Horner syndrome.
  • On examination, he had a right abduction deficit consistent with an abducens nerve palsy.
  • The combination of an abducens nerve palsy and a Horner syndrome indicated a cavernous sinus localization. The history of slow progression over months indicated a compressive or infiltrative lesion.
  • The proximity to critical neurologic structures makes it difficult for any invasive procedures like biopsy or excision.
How well did you know this?
1
Not at all
2
3
4
5
Perfectly
35
Q

A 20-year-old college student from Illinois was admitted for chronic headaches for the past 4 months. She has no travel history or occupational exposure to toxins or infectious agents. Her vital signs are stable. The physical exam is remarkable for lymphadenopathy, erythema nodosum, ataxia, and right facial droop without facial sparing. An MRI with gadolinium shows abnormal signal intensities in the cerebellum and brainstem. A lumbar puncture reveals elevated protein and mild pleocytosis. If a lymph node biopsy is done, what is the most likely histopathology that would be seen?

A
  1. Encapsulated yeast
  2. Caseating granulomas
  3. Fibrosis
    4. Non-caseating granulomas

  • This patient has neurosarcoidosis. Non-caseating granulomas are the hallmark of sarcoidosis.
  • A lumbar puncture will show elevated protein and mild-to- moderate pleocytosis.
  • An MRI with contrast may show leptomeningeal enhancement.
  • A lumbar puncture may help exclude infection of the central nervous system, malignancy, or multiple sclerosis.
How well did you know this?
1
Not at all
2
3
4
5
Perfectly
36
Q

A 32-year-old female has been experiencing visual deficits over the last week. She reports that she does not believe her family history includes any major eye conditions. Her extraocular muscle function, intraocular pressure, fundoscopy results, and pupillary reflex are normal. During a confrontation visual field test, the patient demonstrates a wedge-shaped, right homonymous hemianopsia. Which of the following arteries would most likely be obstructed?

A
  1. Posterior pericallosal branch of the posterior cerebral artery
  2. Central retinal branch of the ophthalmic artery
    3. Lateral posterior choroidal branch of the posterior cerebral artery
  3. Supraorbital branch of the ophthalmic artery

  • The lateral posterior choroidal branch of the posterior cerebral artery vascularizes the medial and posterior portions of the lateral geniculate nucleus (LGN).
  • The only other vascularization of the lateral geniculate nucleus is from the anterior choroidal artery.
  • Arteriovenous malformation of the lateral geniculate nucleus can produce homonymous hemianopsia.
  • Damage to the LGN may present without concomitant effects on visual feedback responses, such as some forms of extraocular muscle function or pupillary reflex.
How well did you know this?
1
Not at all
2
3
4
5
Perfectly
37
Q

A 19-year-old male is brought to the trauma bay after being found by his roommate shortly after a suicide attempt by hanging himself in their dorm room. On arrival, the patient is hemodynamically stable and neurologically intact. He is placed in a rigid cervical collar in transport, and he is taken to the CT scanner on arrival. A non-displaced bilateral C2 pars interarticularis fracture with less than 3 mm subluxation of C2 on C3 is reported by the radiologist. The family asks about the prognosis of his injury and plan of care. What is the most appropriate response?

A
  1. His fracture can be treated with a rigid cervical collar for at least four weeks, but we will need to acquire further imaging studies to evaluate his vascular structure and the ligaments in his neck, and he will most likely need surgical fixation as well
  2. His fracture will require immediate surgical intervention with permanent fixation
    3. His fracture can be treated with a rigid cervical collar for at least eight weeks, but we will need to acquire further imaging studies to evaluate his vascular structure, and the ligaments in his neck and the fracture has a high chance of healing on its own
  3. His fracture can be treated with a rigid cervical collar for at least six weeks, but we will need to acquire further imaging studies to evaluate his vascular structure, and the ligaments in his neck and the fracture has a high chance of healing on its own

  • A fracture of the bilateral pars interarticularis of C2 is also known as a Hangman fracture.
  • Hangman fracture without significant subluxation or angulation has a greater than 90% chance of healing on its own and should be treated with a rigid cervical collar for at least eight weeks.
  • It is important to obtain a CT angiogram of the neck to evaluate for vertebral artery occlusion or dissection, which could result in more serious neurologic damage.
  • Additionally, an MRI of the cervical spine is necessary as damage to the ligamentous structures or the C2-3 disc space may require surgical intervention.
How well did you know this?
1
Not at all
2
3
4
5
Perfectly
38
Q

A 46-year-old man presents to the clinic after being referred by his palliative medicine specialist. The patient has a history of metastatic lung cancer with debilitating pain severely impacting his quality of life. Despite aggressive medical management and pain specialist intervention, his pain persists, and he reports that he is starting to experience thoughts of suicidal ideation because of his discomfort. The patient requests to be provided with information about a well-known invasive surgical procedure, which is often performed in patients with cancer with refractory somatic pain. Which of the following is the most common complication leading to mortality in patients that undergo this procedure?

A
  1. Cardiac failure
    2. Respiratory failure
  2. Liver failure
  3. Kidney failure

  • Respiratory dysfunction and failure is the most common complication leading to mortality in cordotomy procedures.
  • The mortality rate among cordotomy patients is documented as high as 6%.
  • With current improved techniques and advancements in ablation devices, rates of respiratory failure and dysfunction are progressively decreasing.
  • Poor respiratory function before undergoing percutaneous cervical cordotomy has not been shown to increase rates of respiratory failure.
How well did you know this?
1
Not at all
2
3
4
5
Perfectly
39
Q

A 49-year-old male presents for an initial consultation having been referred by her primary care provider for low back and bilateral radicular pain present for the past 4 months. His left sided leg pain has responded to formal physical therapy but his right sided leg pain has now worsened. A trial of NSAIDs did not alleviate his pain. He now also notices some difficulty in maintaining a normal gait for long distances. His physical exam is unremarkable except for 4/5 strength in plantarflexion of the right ankle and some subjective numbness on the dorsum of the foot. Flexion and extension standing plain films do not demonstrate any instability or listhesis. An MRI is obtained which demonstrates a left-sided far lateral disc herniation at L5/S1 as well as a right sided paracentral disc herniation at L5/S1. What is the best next step in the management of this patient?

A
  1. Physical therapy to focus specifically on lower core strengthening and Right lower extremity strength
    2. L5/S1 right paracentral discectomy
  2. L5/S1 right paracentral and left far lateral discectomies
  3. L5/S1 subtotal discectomy with L5/S1 interbody fusion

  • Generalized formal physical therapy should be trialed in patients with symptomatic lumbar disc herniations, however, if failed, other treatment options should be considered. This patient has failed a trial of formal physical therapy, and his pathology is not indicative of predicted responsiveness to any demonstrated specific therapy protocol. Surgical intervention has a higher likelihood of providing a positive outcome.
  • This patient has a symptomatic lumbar disc herniation, as regarding his right-sided symptoms has failed a trial of conservative therapy. Surgery in the form of discectomy is indicated at this time as it has a relatively higher likelihood of positive outcome when compared to continued nonoperative therapy. Despite pathology at on the left side at L5/S1, symptoms resulting from this specific disc herniation have resolved with physical therapy. When performing discectomy, surgical intervention should be focused only on correlated clinical and radiographic findings to maximize outcome and preserve segmental stability. As such, right paracentral discectomy at L5/S1 is the most appropriate treatment option.
  • Strict and demonstrable associated clinical and radiographic evidence of pathology should be pursued when planning discectomy. As the symptoms caused by the left-sided far lateral herniation at L5-S1 have resolved and there are no abnormalities on physical exam of the left side, this herniation should not be addressed surgically to maintain segmental stability in the setting of the right-sided partial discectomy for the symptomatic paracentral herniation.
  • Lumbar fusion is not indicated in the setting of symptomatic disc herniation, despite multiple foci of pathology. This patient’s work up is notable for the absence of instability on x-ray. L5/S1 right sided paracentral discectomy is the indicated surgical option to address this patient’s pathology.
How well did you know this?
1
Not at all
2
3
4
5
Perfectly
40
Q

A 54-year-old female with past medical history significant for polycystic kidney disease presents to the emergency department by ambulance after developing sudden onset severe headache and neck stiffness that started 2 hours ago. On exam, the patient is awake and alert but uncomfortable due to pain. She is noted to have a Cranial nerve III palsy on the left but otherwise normal neurologic exam. Vitals are blood pressure 170/106 mmHg, pulse 100 bpm, temperature 97.6 F, respiratory rate 18/minute, oxygen saturation 100% on room air. CT scan reveals subarachnoid hemorrhage. Based on the Hunt and Hess classification system, what grade of subarachnoid hemorrhage does this patient have?

A
  1. 1
    2. 2
  2. 3
  3. 4

  • This patient presents with sudden onset, severe headache, and neck stiffness. This is concerning for subarachnoid hemorrhage (SAH) from a ruptured cerebral aneurysm.
  • This patient has a history of polycystic kidney disease, which is a risk factor for cerebral aneurysm formation, and ultimately, rupture.
  • The Hunt and Hess Classification system can be used to grade patients with subarachnoid hemorrhage from ruptured cerebral aneurysm. The system uses initial neurologic status to determine grade, which can be used to predict overall mortality.
  • This patient presented with a severe headache, neck stiffness, and a left cranial nerve III palsy. This is consistent with a grade 2 on the Hunt and Hess Classification system.
How well did you know this?
1
Not at all
2
3
4
5
Perfectly
41
Q

Which class of medication is indicated for the treatment of moderate to severe pain, but caution must be exercised in using them to treat post-craniotomy headaches secondary to their addictive potential, ability to cause respiratory depression, and capability of obscuring neurologic responses?

A
  1. Glucocorticoids
  2. Non-steroidal anti-inflammatory drugs
    3. Opioids
  3. Anti-epileptic drugs

  • To decrease use of opioids postoperatively, one single center, randomized, blinded trial studied preoperative diclofenac as a prophylactic agent. It decreased the intensity of headache on postoperative day 1, and up to post-op day 5 for patients with infratentorial surgeries. However, there always remains a risk of bleeding with the use of non-steroidal anti-inflammatory medications.
  • Long-term use of opioids is not indicated in the treatment of chronic post-craniotomy headaches. One alternative approach is the use of botulinum toxin A, which works presynaptically to block the release of acetylcholine at the neuromuscular junction.
  • Of particular concern with opioid administration is the possibility of respiratory depression. Therefore, research is now focusing on medications like dexmedetomidine, which is an alpha-2 adrenergic agonist that has the potential for treatment of post- craniotomy headaches and is a sedative agent that does not carry risk of respiratory depression.
  • In post-craniotomy patients, opioids may obscure physical exam findings and red flag signs that may indicate urgent or emergent work up. Differential diagnoses to keep in mind post-operatively include cerebrospinal fluid leak, intracranial hemorrhage, and meningitis or infection.
42
Q

A 45-year-old patient presents to the office complaining of low back pain after heavy work on his farm. No acute trauma is reported. He does, however, state that he used to wrestle when he was young and has suffered from low back pain since high school. The patient does not report leg pain. Anterior-posterior and lateral lumbosacral X-rays reveal a Mayerding grade III spondylolisthesis at L5-S1. A lateral X-ray shows a lucency in the region of the pars interarticularis with signs of sclerosis in the borders of the defect. Which type of spondylolisthesis is the most probable type, according to Wiltse?

A
  1. Type I: dysplastic
    2. Type II: isthmic
  2. Type III: degenerative
  3. Type IV: traumatic

  • Injury of the pars interarticularis is one of the most common identifiable causes of ongoing low back pain in adolescent athletes; sometimes, this condition is not detected until adulthood, when the injury becomes chronic and symptomatic.
  • The vast majority of pars defects, also called spondylolysis, occur at L5 (85–95%), with L4 being the next most commonly affected level (5 to 15%).
  • Although lateral lumbosacral radiographs may show a defect in pars in 80%, the classical oblique X-ray views have been mentioned as essential studies to diagnose this condition.
  • Type I: dysplastic: congenital abnormalities of L5 or the upper sacrum allow anterior displacement of L5 on the sacrum. Type III: degenerative: secondary to long-standing intersegmental instability with the associated remodeling of the articular processes. Usually, in elderly patients. Type IV: traumatic: acute fractures in the vertebral arch other than the pars.
43
Q

A 24-year-old patient was planned for operative management to remove a large intraventricular tumor with evolving obstructive hydrocephalus. Following the opening of the dura, the brain was tight, and slowly the brain started mushrooming out of the craniotomy site despite the administration of mannitol. What would be the next best step in managing this intraoperative issue?

A
  1. Enlarge the craniotomy
  2. Rapidly close the brain, keep the patient paralyzed, and start furosemide
    3. Attempt to tap the ventricle
  3. Remove the herniated brain tissue and approach the tumor

  • Intraoperative brain swelling can be managed with an attempt to tap the ventricles and thereby allow egress of some CSF.
  • Paine’s point can be approached intraoperatively during the intraoperative period.
  • The point of the ventricular tap is through the apex of an isosceles triangle, whose base is 3.5 cm along the Sylvian fissure, and each of its limbs is 2.5 cm. This can lead to temporary laity of the brain, providing time for targeting and then debulking the tumor.
  • Closing the brain after removing the herniated brain and then keeping the patient ventilated will not prevent the secondary damage and is only undertaken as a last resort in managing such an emergency.
44
Q

A 78-year-old female presents to the emergency department after waking up in the morning with severe midback pain. She denies any trauma. She lives alone. On clinical examination, she is obese with a BMI of 42 and has mid-thoracic midline tenderness but is neurologically intact. A CT scan of her thoracic and lumbar spine demonstrates a fracture at T8 with a 50% loss of the anterior vertebral body with no extension in the posterior columns. There is no kyphosis or any canal compromise. The patient has persistent pain refractory to intravenous analgesics and hampering her daily activities. Which is the most reasonable next step in managing this patient?

A
  1. TSLO brace mobilization
    2. Cement vertebroplasty
  2. Mono-segment posterior pedicle screw fixation
  3. Long segment posterior pedicle screw fixation

  • Given the description of the CT findings, the patient has a low thoracic compression fracture with moderate loss of vertebral body height.
  • Cement augmentation will allow for stabilization of fracture, decreased pain, and increase functionality quickly for this patient. It can be performed as a percutaneous daycare surgery.
  • She lives independently, so a means of treatment that allows her to improve her functionality and quality of life quickly, such as cement augmentation, is a good choice for her.
  • While there is level 5 evidence to suggest mobilization without bracing is ok, the published literature supports bracing or cement augmentation as opposed to no immobilization (answers 1 and 4). Based on the clinical vignette, this patient will likely not succeed with an extended period of bracing due to body habitus and pain level. This patient is obese and will not be adequately immobilized with a TLSO. Therefore, cement augmentation will help with improving her pain, function, and quality of life. Pedicle screw fixation is a major procedure and is only advocated in unstable fractures, and in patients with canal compromise.
45
Q

A 57-year-old male with a past medical history significant for diabetes and chronic sinusitis is scheduled for a sphenopalatine ganglion radiofrequency ablation utilizing the infra zygomatic approach. After the radiofrequency (RFA) needle is placed in the final position, stimulation confirms proper needle positioning. On injection of a local anesthetic to anesthetize the ablation area the patient complains of liquid running into the back of his throat. What step(s) should the interventionalist take at this point?

A
  1. The RFA needle withdrawn, and the procedure aborted; the patient should be monitored for an infection or bleeding.
  2. The RFA needle withdrawn and the procedure aborted and an urgent ENT consult should be placed to evaluate for any structural damage to the nasopharyngeal area.
    3. The RFA needle should be withdrawn slightly and retested; if proper stimulation is achieved, local anesthetic should be injected to anesthetize the area; if the patient does not report a liquid feeling in the back of his throat the radiofrequency ablation should be completed, and the patient should then be monitored for any symptoms such as bleeding or infection.
  3. The RFA needle should be withdrawn slightly and retested; if proper stimulation is achieved, local anesthetic should be injected to anesthetize the area; if the patient does not report a liquid feeling in the back of his throat the radiofrequency ablation should be completed, and post-procedure the patient should be placed on broad-spectrum oral antibiotics and monitored for an infection or bleeding.

  • The sensation of liquid running down the patient’s throat is an indication that the nasopharyngeal space has been entered.
  • If the needle enters the nasopharyngeal space, the clinician should monitor the patient for bleeding and infection. The patient should also be educated on which signs and symptoms to be aware of and report.
  • Although aborting the procedure is an option, in most cases it is unnecessary as the procedure can be completed safely.
  • After puncture of the nasopharyngeal cavity starting the patient empirically on antibiotics is not recommended as this could lead to an antibiotic-resistant infection.
46
Q

A 17-year-old male is brought into the emergency department by paramedics after jumping off a two-story building. When found, he was unresponsive on a cement parking lot. On arrival to the emergency department, the Glasgow coma scale is 6 and the patient is immediately intubated for airway protection. On initial evaluation, there is a large area of ecchymosis behind his right ear. Which of the following sets of vital signs indicates impending brain herniation?

A
  1. A heart rate of 130 beats per minute, blood pressure of 90/50 mmHg, and respiratory rate of 30 breaths per minute
  2. A heart rate of 115 beats per minute, blood pressure of 90/50 mmHg, and respiratory rate of 22 breaths per minute
  3. A heart rate of 70 beats per minute, blood pressure of 205/120 mmHg, and respiratory rate of 23 breaths per minute
    4. A heart rate of 48 beats per minute, blood pressure of 165/70 mmHg, and respiratory rate of 10 breaths per minute

  • Cushing reflex is a physiological response to increased intracranial pressure, which is seen in severe traumatic brain injury.
  • Cushing triad is a late sign of impending herniation.
  • Cushing triad includes bradycardia, hypertension, and respiratory depression. In children, it can cause either bradycardia or tachycardia.
  • The presence of Cushing triad is an ominous condition and requires urgent evaluation and subsequent intervention.
47
Q

A 69-year-old male presents with a right hemispheric cerebral stroke. Ultrasound reveals a 90% carotid bulb stenosis. He has also had a myocardial infarction three weeks before experiencing this stroke and has a history of oxygen-dependent chronic obstructive pulmonary disease. CT scan of the chest reveals a type 3 aortic arch and severe aortic calcifications. He is recovering as expected from his stroke. What is the next best intervention?

A
  1. Transfemoral carotid artery stent
  2. Carotid artery endarterectomy
    3. Carotid artery stenting
  3. Repeat carotid CT angiography after 6 months

  • It is reasonable to intervene for asymptomatic carotid artery stenosis in 2 to 3 weeks of the cerebrovascular event if the patient is recovering from stroke as expected.
  • Type 3 aortic arch and aortic calcifications can make a transfemoral approach for stenting difficult.
  • This patient is a high risk for carotid endarterectomy given his recent myocardial infarction and chronic obstructive pulmonary disease.
  • Transcarotid carotid stenting with flow reversal represents the next best step along with medical management including aspirin, clopidogrel, and a statin.
48
Q

A 27-year-old man is brought to the emergency department unconscious by ambulance after being struck by a motor vehicle while riding a bicycle. Computed tomography demonstrates a temporal bone fracture. Upon awakening, the patient is noted to have ipsilateral complete hemifacial paralysis. What is the best next step in the management of this patient?

A
  1. Computed tomography scan of the head and neck with contrast 2. Electroneuronography
  2. Magnetic resonance imaging of the brain
  3. Blink reflex test

  • Electroneuronography (ENoG) is the gold standard for the evaluation of facial nerve function. The facial nerve can be acutely transected in the setting of a temporal bone fracture.
  • ENoG is a noninvasive study that utilizes brief electrical stimulation and subsequent recording of propagated action potentials. Amplitude is the key parameter that is compared to the contralateral unaffected side of the face.
    ENoG provides excellent prognostic value if used after 3-5 days from injury. Prior to this timeframe, the distal portion of a transected nerve still maintains some stimulability.
  • Because ENoG testing relies on comparison of the injured side of the face to the normal side of the face, it is not applicable for patients with bilateral facial nerve injuries.
  • A computed tomography (CT) scan with contrast would not be able to identify facial nerve damage in this patient, although it could be used to assess for ischemic stroke. Although strokes can cause facial paralysis, in the setting of acute temporal bone fracture, suspicion for facial nerve injury should be high. Blink reflex testing is based on the corneal reflex. The facial nerve serves as the efferent limb of the corneal reflex, and its function can be assessed by this form of testing. This would not provide any significant diagnostic or prognostic information but may be used to monitor recovery progress. Magnetic resonance imaging may be used to evaluate the intracranial contents in the event of unexplained neurological findings, but the CT obtained originally revealed the etiology of the facial paralysis.
49
Q

A 62-year-old female is treated with a ventriculoperitoneal shunt for hydrocephalus. One month later, she presents with a headache, unsteadiness, and fever. She attends the emergency department and discusses the case with the on-call neurosurgical team. Following a normal set of labs, including inflammatory markers and CBC, a shunt tap is performed, and a plain CT head and shunt radiographs are requested. Which of the following is the most accurate statement about the patient’s condition?

A
  1. The CT head is likely to diagnose the condition
    2. The shunt tap is likely to diagnose the underlying condition
  2. This patient needs a lumbar puncture to diagnose the underlying condition
  3. The CT head will show ependymal enhancement and hydrocephalus.

  • CSF in ventriculitis may show a raised cell count and protein, with low glucose.
  • CSF culture may be positive, although this is not always the case, especially if the sample is taken following antibiotic administration.
  • The presence of oligoclonal immunoglobulin G or M bands, CSF lactate, procalcitonin, and lysozymes help make an early diagnosis.
  • Non-contrast CT head demonstrates non-specific findings, including dependent hyperdense ventricular debris, hydrocephalus, periventricular low density as well as features of the underlying abnormality. It does not offer a definitive diagnosis. A CSF sample should be obtained. CSF sample can be obtained via a shunt tap, it does not need to be obtained via lumbar puncture. The patient had a plain CT head. Enhancement may be seen on a contrast-enhanced scan. A CSF sample will also be required. A wound infection needs to be considered - an examination of her abdomen and the cranial wound is imperative. A shunt series radiograph may help rule out a blocked shunt. Comparison of the new and previous CT scans may help decide if the patient has hydrocephalus, which may suggest a blocked shunt.
50
Q

Which of the following is a rare complication in a patient with autosomal dominant polycystic kidney disease?

A
  1. Cardiac disease
  2. Renal stones
    3. Subarachnoid hemorrhage
  3. Urinary tract infections

  • Despite the potential for death, subarachnoid hemorrhage is a very rare occurrence in patients with autosomal dominant polycystic kidney disease.
  • The aneurysms in the brain do not always rupture, as was once believed.
  • The major cause of death in these patients is cardiac disease.
  • Cerebral aneurysms are serious but in fact are very rare. Rupture is most common in patients with uncontrolled hypertension.
51
Q

Following a traumatic injury to the lower arm, a patient is diagnosed with stage two peripheral nerve injury (Axonotmesis). Which of the following is true?

A
  1. The patient has a good prognosis for recovery in weeks, with similar conduction velocity compared to pre-injury
    2. The patient has a good prognosis for recovery within 3-4 months, yet with slower conduction velocity than pre-injury
  2. The patient has a poor prognosis for a full recovery, with similar conduction velocity compared to pre-injury
  3. The patient has a poor prognosis for any functional recovery

  • Neurapraxia is the mildest form of peripheral nerve injury, characterized by local damage to an axon that does not disrupt axonal continuity. Common after a local mechanical compression, a full recovery is usually complete within days to weeks.
  • Axonotmesis is defined by interruption of the axons, but with preservation of the basal lamina connective tissue that allows axonal regeneration. Regeneration occurs at approximately 1 inch per month. Schwann cells will be remyelinated with thin and nodes more closely spaced, thus decreasing conduction velocity.
  • Axonotmesis results in interruption of the axon, but with preservation of Schwann cell basal lamina that allows axonal regeneration. In a more severe injury, with poor recovery, one would observe a decrease in conduction velocity or a conduction block.
  • Neurotmesis, the most severe form of peripheral nerve injury, results in a loss of continuity in both the axon and the surrounding connective tissue. Surgical end-to-end reconnection of the nerve stumps is usually required.
52
Q

A 6-year-old male is brought to the clinic for evaluation. The patient has been developing progressive scoliosis. MR imaging is notable for a thoracolumbar syrinx with a low lying conus medullaris and fatty filum. Surgical correction options are discussed. Which of the following are potential risks that the patient’s physician would discuss with them that could result from surgery?

A
  1. Weakness of the upper extremities
  2. Cerebrospinal fluid (CSF) fistula
  3. Nerve root injury
    4. CSF fistula and nerve root injury

  • Postoperative weakness of the upper extremities would not be considered to be a primary complication following surgery for a closed neural tube defect involving the lumbosacral spine. Surgical intervention can result in nerve root injury or injury to the conus medullaris. Associated deficits would include motor and/or sensory compromise of the lower extremities in addition to the potential for bowel/bladder compromise. Surgical interventions in the lumbosacral region would not be associated with any abnormalities of the upper extremities.
  • Postoperative CSF fistulas would be considered to be a primary complication following surgery for a closed neural tube defect involving the lumbosacral spine. The complication of a CSF cutaneous fistula is best prevented by meticulous dural and fascial closure. Nonabsorble monofilament suture is preferred for these layers.
  • Postoperative nerve root injury would be considered to be a primary complication following surgery for a closed neural tube defect involving the lumbosacral spine. Nerve root damage is avoided by meticulous dissection and the use of intraoperative neurophysiologic monitoring.
  • As noted above, Postoperative CSF fistulas and postoperative nerve root injury would be considered to be primary complications following surgery for a closed neural tube defect involving the lumbosacral spine. Wound infection would also be considered to be a primary complication following surgery for a closed neural tube defect involving the lumbosacral spine. Wound infection is best addressed in the infant/toddler population by frequent scheduled diaper changes and aggressive family education as to proper wound care.
53
Q

A 51-year-old man is admitted to the hospital for severe traumatic brain injury. The patient is found unresponsive 48 hours after admission. Physical examination shows loss of brain stem reflexes. An apnea test is aborted because the patient develops hypoxia. A decision is made to perform an EEG to confirm brain death. Which of the following best helps decrease the likelihood of a false-positive test?

A
  1. Increase the EEG sensitivity to 4 microvolts/mm
    2. Add electrocardiogram (ECG) monitoring
  2. Administer midazolam before the test
  3. Interelectrode distances should be less than 5 centimeters

  • Concomitant electrocardiogram (ECG) monitoring is essential to detect any ECG artifacts in the EEG.
  • EEG can confirm brain death when it shows no electrical activity of more than 2 microvolts, at a sensitivity of 2 microvolts/mm for at least 30 seconds. EEG sensitivity should be at least 2 microvolts/mm to assess electrocerebral inactivity more accurately.
  • It should show no reactivity to intense somatosensory stimuli.
  • Sedation should not be given to the patient as it may affect the EEG. Interelectrode distances should be at least 10 centimeters during testing for electrocerebral inactivity.
54
Q

A 74-year-old man comes into your office complaining of constant low back pain. He has a history of a previous spinal fusion at T12/L1. Which of the following parameters would lead you to suspect sagittal imbalance as the cause of the pain in the patient?

A
  1. Sagittal vertical axis (SVA) of 4.8 mm
  2. Pelvic incidence of 62 degrees
    3. 20 degrees of kyphosis at the thoracolumbar junction
  3. Lumbar lordosis of 55 degrees

  • Sagittal imbalance plays a critical role in the causation of a spinal instability and the genesis of chronic low back pain and failed-back surgery syndrome.
  • The thoracolumbar junction is normally neutral in the sagittal plane, as it is a transition zone from thoracic kyphosis to lumbar lordosis.
  • Twenty degrees of kyphosis at this region is too much kyphosis and indicates that sagittal imbalance is present.
  • The sagittal vertical axis, lumbar lordosis, and pelvic incidence are within normal reference values in this patient.
55
Q

A 64-year-old patient with alcohol use disorder lives alone and is admitted for a left leg wound that has failed to heal. He is edentulous, has multiple ecchymoses, minute hemorrhages around hair follicles, and splinter hemorrhages in the nail beds. What should be given to the patient?

A
  1. Vitamin A
    2. Vitamin C
  2. Vitamin B
  3. Vitamin K

  • Deficiency of dietary vitamin C is called scurvy.
  • Impaired hydroxylation of proline residues in the nascent procollagen chains leads to weakness of blood vessel walls and is caused by the deficiency of vitamin C.
  • Scurvy is characterized by splinter hemorrhages, perifollicular hemorrhages, fragmentation of hairs, purpura, ecchymoses, and hemorrhages into muscle.
  • Ascorbic acid is needed for biosynthetic pathways by accelerating hydroxylation and amidation reactions. In the synthesis of collagen, ascorbic acid is required as a cofactor for prolyl hydroxylase and lysyl hydroxylase. These enzymes are responsible for the hydroxylation of the proline and lysine amino acids into collagen. Hydroxyproline and hydroxylysine help stabilize collagen by cross-linking the propeptides in collagen. Defective collagen fibrillogenesis results in impairment of wound healing. Collagen is part of a bone, so bone formation is affected. Defective connective tissue leads to fragile capillaries, resulting in abnormal bleeding. Untreated scurvy is invariably fatal.
56
Q

A 70- year-old male patient presents to the emergency department with recurrent dizziness and fall incidents. Right carotid auscultation revealed moderately to severe bruit. Doppler sonography elucidated 80% stenosis with peak systolic velocity: 300 cm/sec in the right internal carotid artery. What is the preferred next step in the management of the patient?

A
  1. Aspirin
  2. Bypass surgery
  3. Carotid artery stenting
    4. Open endarterectomy

  • Currently, the society for vascular surgeons recommends carotid endarterectomy as first-line treatment for most symptomatic patients with stenosis of 50% to 99% and asymptomatic patients with stenosis of 60% to 99%.
  • The risk of stroke is less with carotid endarterectomy compared to carotid artery stenting. The elevated risk of myocardial infarction in carotid endarterectomy in comparison with stenting may be related to the hemodynamic stress induced on the body by clamping a major artery in the body - this is an essential step in the endarterectomy procedure.
  • The inferiority of carotid artery endarterectomy in terms of a higher rate of myocardial infarction was not constant among two recently published randomized controlled trials, the carotid revascularization endarterectomy versus stent trial (CREST) and the international carotid stenting Study (ICSS).
  • Carotid artery stenting should be reserved for symptomatic patients with stenosis of 50% to 99% at high risk for carotid endarterectomy for anatomic or medical reasons. Historically, DSA has been the standard gold test to evaluate the extra- and intracranial circulation. This is an invasive procedure, typically performed via a transfemoral puncture, and involves selective imaging of the carotid and vertebral arteries using iodinated contrast. The risk of stroke during cerebral angiography is generally reported at approximately 1%. It is typically due to chemoembolization related to wire and catheter manipulation in the arch aorta or proximal branch vessels.
57
Q

A 59-year-old female with a history of an indirect carotid cavernous fistula presents to the emergency department for worsening ocular pain and vision loss. The patient had two prior unsuccessful attempts at transvenous embolization due to increased tortuosity of the vessels. Which of the following is the most appropriate intervention for this patient?

A
  1. Radiosurgery
  2. Repeat attempt at transvenous embolization
    3. Direct cannulation of the superior ophthalmic vein following surgical exposure
  3. Conservative management with cervical internal carotid artery pressure

  • The patient presents with an indirect carotid cavernous fistula and worsening symptoms, which requires intervention. Observation and conservative management would not be appropriate because her symptoms may further worsen and lead to vision loss.
  • Another attempt at transvenous embolization is unlikely to be successful if two prior attempts have been unsuccessful.
  • Radiosurgery is not an option for emergency cases because obliteration of the fistula may take weeks to months.
  • Direct cannulation of the superior ophthalmic vein is an alternative when standard transvenous access to the cavernous sinus through the inferior petrosal sinus is not successful.
58
Q

You are consulted for management of a 63-year- old female who has been admitted for signs and symptoms suspicious of vertebral osteomyelitis. Initial blood cultures are negative, and the patient has no signs of sepsis or neurologic impairment. What are your initial recommendations regarding antibiotic therapy in the patient?

A
  1. Begin therapy with broad-spectrum antibiotics immediately and continue to obtain serial blood cultures
  2. Begin therapy with broad-spectrum antibiotic immediately and refer the patient for image-guided biopsy of the affected vertebra
    3. Withhold antibiotic therapy until positive cultures are obtained to guide the antibiotic therapy
  3. Start antibiotic therapy after blood is drawn for cultures

  • The Infectious Diseases Society of America (IDSA) guidelines recommend that in the non-emergent setting (i.e., no signs of sepsis or neurologic compromise) antibiotic therapy can be withheld for up to 1 to 2 weeks.
  • Every attempt is to be made to obtain positive blood or tissue cultures to identify an offending pathogen and appropriately narrow down antibiotic coverage.
  • Numerous intravenous therapeutic regimens are recommended by the IDSA for the treatment of vertebral osteomyelitis with options for oral therapies as well.
  • Antibiotic stewardess as per the culture and sensitivity report is of paramount impotence to prevent antibiotic resistance.
59
Q

A patient has been complaining of decreased sensation in the lower extremities. Laboratory studies reveal a hemoglobin 9.8 g/dL, hematocrit 33.2%, and MCV 125 fL. Microscopic examination of peripheral blood smear shows red blood cells with macrocytosis and neutrophils with hypersegmentation of nuclei. Which of the following is the most likely deficiency?

A
  1. Vitamin C
  2. Ascorbic acid
  3. Iron
    4. Cobalamin

  • A diagnosis of pernicious anemia is made through blood work. Patients usually have a high MCV and a normal MCHC.
  • Ovalocytes are present on the smear with hypersegmentation of the neutrophils.
  • The patient may complain of a triad of paresthesias, sore tongue, and weakness. Impaired urination, loss of sensation in feet, unsteady gait, and clumsiness are common in chronic cases.
  • Repletion of cobalamin is the treatment.
60
Q

A 17-year-old Asian male recently stopped playing guitar because of “poor playing,” as reported by his band. He also reports difficulty to button, typing, and texting with his right hand. He comes to the office with right-hand thenar eminence atrophy, decreased grip strength, and decreased fine motor and object manipulation in the right hand. The power of his wrist extension is 3/5, and his supinator reflex is 2+/4. The right elbow and shoulder strength are 5/5. Left upper extremity is 5/5 on manual muscle exam. Bilateral sensory is intact. The dynamic MRI cervical spine showed anterior displacement of the cord. Which of the following is the most rational next step in managing the patient?

A

1. Cervical collar application
2. Cervical laminoplasty alone
3. Cervical posterior implant fixation
4. Duraplasty with tenting

  • This patient is demonstrating a classical description of Hirayama disease. While often considered a benign or self-limiting condition with a plateau phase, an increased rate of progression may sometimes be observed.
  • Conservative management includes therapy to maintain function and strength until the plateau phase of the process is reached. Eary initiation of therapeutic intervention is essential to maintain independence and limit functional deficits.
  • A neck collar promotes cervical spine stability and avoids further cervical spine atrophy by decreasing microtrauma events.
  • Surgical management is reserved for patients with rapid progression, extreme limitations in function, or bilateral involvement. This procedure is known to have good results. However, patients may have a limited return of lost functions.
61
Q

A 45-year-old male patient has been scheduled for clipping for an anterior communicating artery (ACOM) aneurysm. The identification and preservation of the recurrent artery of Heubner (RAH) are of paramount importance so as to minimize associated neurological sequelae. What is the most reliable cue to correctly identify the RAH intraoperatively?

A
  1. The first division of the anterior cerebral artery
    2. The recurrent course along the first anterior cerebral artery branch (A1) towards the anterior perforated substance
  2. Its origin from the A1-ACOM junction
  3. Its communication to other lenticulostriate vessels

  • The RAH has a characteristic course following its origin, which can be of help for its identification intraoperatively.
  • It always follows a recurrent course along with the first division of the anterior cerebral artery (A1) while transiting towards the anterior perforated substance.
  • This course is therefore responsible for its latest naming as the Recurrent artery of Heubner.
  • Orbitofrontal artery is the first division of the anterior cerebral artery. The origin of RAH is variable and can arise from A1, A1- ACOM junction, or A2. It sometimes only form rete communication with branches from other lenticulostriate vessels.
62
Q

Optic nerve sheath fenestration (ONSF) should be considered in which one of the following scenarios?

A
  1. An overweight 25-year-old woman who presents with severe chronic headaches with normal vision and full visual fields with no optic disc edema and an opening pressure of 35 cm H2O
  2. A 45-year-old man with a malignant pineal region tumor causing hydrocephalus with bilateral 20/25 vision and grade 2 disc edema with mild blind spot enlargement
  3. A 25-year-old woman who presents with chronic headaches, pulsatile tinnitus, transient visual obscurations, mild bilateral optic disc edema, and no visual field defects with a diagnosis of pseudotumor cerebri (PTC) who has not received any treatment
    4. An overweight 25-year-old woman who presents with severe bilateral vision loss, headaches, pulsatile tinnitus, and field constriction with severe bilateral optic disc edema, and an opening pressure of 55 cm H2O by lumbar puncture

  • It is important to ensure that patients meet the diagnostic criteria for the diagnosis of PTC syndrome prior to considering surgical intervention. In patients, without optic disc edema or a localizing neurologic deficit the diagnosis of PTC syndrome can only be considered. ONSF is not indicated in the absence of optic disc edema and is not a primary treatment for headache management.
  • In the setting of secondary intracranial pressure (ICP) elevation from tumors, hydrocephalus, metabolic derangements, venous sinus thrombosis, etc., every effort should be made to directly address the underlying cause of ICP elevation as visual loss from papilledema is secondary to these etiologies. In the setting of brain tumors causing ICP elevation appropriate surgical and medical management should be instituted to address the lesion.
  • In patients with malignant PTC and severely elevated ICP causing severe vision loss, both ONSF and cerebrospinal fluid diversion procedures are sometimes performed concomitantly in an attempt to maximize the chance of preserving vision. This requires communication between ophthalmology and neurosurgery regarding the severity of the case and coordination of surgical procedures.
  • This patient has malignant PTC syndrome and is a candidate for ONSF as she has symptoms and signs of ischemic end-organ damage to the optic nerves from severely elevated ICP.
63
Q

A 57-year-old female presents to the emergency department with a chief complaint of saddle anesthesia of the left inner thigh and paresthesias in the left lower extremity. She reports no previous episodes and denies a history of trauma. Physical examination reveals a diminished left patellar reflex. The heart and lungs are clear to auscultation, and her gait is abnormal. An MRI of the spine is most likely to reveal which abnormality?

A

1. Herniated nucleus pulposus
2. Sacral fracture
3. Sciatic nerve compression
4. Conus medullaris lesion

  • Cauda equina lesions which may affect the patellar and Achilles reflexes can be distinguished from lesions to the conus medullaris, which affect the Achilles reflex but spare the patellar reflex.
  • Cauda equina lesions typically involve a herniated nucleus pulposus but also may occur due to a tumor or abscess.
  • The nucleus pulposus typically herniates in the postero-lateral direction due to the weak nature of the posterior longitudinal ligament.
  • Cauda equina syndrome may occur slowly over time but should be quickly addressed to prevent permanent sequelae.
64
Q

A 12-year-old girl is brought to the hospital following the weakness of both upper and lower limbs following a trivial fall injury. She has a short and webbed neck on physical examination with restricted neck movements. There is a weak gag reflex on neurological examination. Radiological imaging revealed that her odontoid tip was above the Chamberlain line, McRae line, and Wackenheim line and was compressing on the brainstem. Which of the following best describes the commonly accepted pathophysiology of the condition in the child?

A
  1. Crowded posterior fossa
  2. Embryological dysgenesis
  3. Mechanical dysgenesis
    4. Atlantoaxial facet instability

  • This is a variant of Group A Basilar Invagination.
  • Atlantoaxial facet instability predisposes to odontoid peg migration through the foramen magnum thereby causing brainstem compression.
  • All other associated anomalies such as the short neck, torticollis, Chiari malformation and syringomyelia is suggested to be secondary to basilar invagination only.
  • Reversal of this facetal slip and craniovertebral fusion has been shown to reverse such pathological sequelae and improve neurological outcome among patients. Crowded posterior fossa predisposes to Chiari malformations.
65
Q

A 26-year-old Asian male presents to the emergency department (ED) because of the sudden onset of aphasia, slurring of speech, and some right facial droopiness. He reports that it lasted about 10 minutes and subsided spontaneously. He has had several similar episodes over the past 2 months. Examination in the ED reveals normal vital signs and a normal neurologic exam. A head CT is normal. A CT angiogram shows bilateral, terminal internal carotid artery occlusion with the presence of moyamoya vessels. What is the best next step in management?

A
  1. Start an antihypertensive drug and a statin
  2. Start aspirin and clopidogrel to prevent future strokes
  3. Start oral warfarin with heparin bridging
    4. CT perfusion studies

  • The patient presented with recurrent ischemic events, which are pathophysiologically due to hypoperfusion.
  • Regular therapies used for stroke prevention, including antihypertensives, statins, and antiplatelets are not effective for moyamoya disease.
  • Anticoagulation is not effective and may cause an increased risk of bleeding.
  • It is important to perform perfusion study and acetazolamide challenge to identify the perfusion deficits and reserves, which will help in making decisions regarding revascularization therapy.
66
Q

A 17-year-old male presents to the emergency department with a 2-day history of fevers, headache, photophobia, and neck pain. The patient is 14 days status post craniotomy for a gunshot wound to the head. On exam, the patient is lethargic but oriented to person, place, and time. There is neck stiffness, but no focal deficit can be found. Workup is significant for a white blood cell count of 26,000 cells/microL and a lactate of 5.2 mmol/L. What should be the initial antibiotic therapy for this patient?

A

1. Vancomycin 20 mg/kg and cefepime 2 g intravenously
2. Vancomycin 20 mg/kg and ampicillin 2 g intravenously
3. Vancomycin 20 mg/kg and piperacillin-tazobactam 4.5 g intravenously
4. Vancomycin 20 mg/kg, ceftriaxone 2 g, and ampicillin 2 g intravenously

  • Cefepime is a fourth-generation cephalosporin with good
  • Pseudomonas coverage. It is recommended in the setting of meningitis with recent neurosurgery or penetrating head trauma. Its structure allows better and more rapid penetration through the cell wall of aerobic gram-negative bacilli.
  • Ceftriaxone can also be used in basilar fractures at a dose of 2 g IV twice a day.
  • Ampicillin is primarily used for Listeria coverage. Listeria should be suspected in neonatal and older populations.
67
Q

A 34-year-old woman presents with disorientation after she had a seizure. Her husband says that she had a low-grade fever three days ago. She has no past medical history. Vital signs show oxygen saturation of 98% on room air, respiratory rate of 20 per minute, heart rate of 110 beats per minute, blood pressure of 110/70 mmHg, and temperature of 102.9 F. On examination, she can move all four extremities spontaneously, and deep tendon reflexes are 3+ without clonus. Plantar reflexes are outgoing bilaterally. The optic disc shows early papilledema, and there is mild neck stiffness. A CT scan of the head is unremarkable. Lumbar puncture shows clear fluid with an opening pressure of 280 mm of water. Cerebrospinal fluid (CSF) analysis shows glucose 77 mg/dL and serum glucose 110 mg/dL. CSF protein 170 mg/dL. There are 100 leukocytes per microL with 85% lymphocytes. There are no red blood cells. What is the most likely causative agent for this patient’s presentation?

A
  1. Gram-negative diplococci
    2. Double-stranded DNA virus
  2. Gram-negative rod
  3. Pathogenic yeast

  • The patient has signs and symptoms of encephalitis, including fever, headache, papilledema, neck stiffness, and a tonic-clonic seizure. She also has an increased opening pressure of CSF with a spinal tap.
  • The CSF spinal tap findings in this patient suggest a viral cause due to an elevated CSF protein level, elevated leukocytes, of which 85% are lymphocytes and a normal CSF glucose concentration. These findings rule out bacterial or fungal causes and indicate a viral cause of encephalitis.
  • The most typical cause of viral encephalitis is herpes simplex encephalitis. Herpes simplex virus is a double-stranded linear DNA molecule.
  • Herpes simplex encephalitis is an acute or subacute illness associated with focal or global cerebral dysfunction caused by herpes simplex viruses belonging to either type 1(HSV-1) or type 2 (HSV-2).
68
Q

A 65-year-old patient presents with complaints of fatigue, headache, and ongoing confusion for the past eight months. His past medical history is significant for several chronic disorders, including diabetes, gout, and arthritis. He has smoked heavily all his life. CT scan reveals multiple aneurysms less than 0.5 mm in the lenticulostriate vessels of the basal ganglia. With what disease process are these aneurysms frequently associated?

A
  1. Advanced age
    2. Hypertension
  2. Von Hippel-Lindau syndrome
  3. Marfan syndrome

  • Charcot-Bouchard aneurysms are microaneurysms that measure 0.8-1.0 mm in diameter.
  • These tiny aneurysms occur at well-defined anatomical locations like the pons, basal ganglia, subcortical white matter, and cerebellum.
  • Charcot-Bouchard aneurysms are associated with long-standing hypertension. Over time, the tiny blood vessels are infiltrated with hyaline and fatty substances, a pathological process that is known as hypertensive lipohyalinosis.
  • Charcot-Bouchard aneurysms are the principal cause of primary intracerebral hemorrhage and commonly arise at the bifurcation of small arteries that lie deep within the brain parenchyma
69
Q

A 46-year-old male presents with complaints of seeing flashing lights, floaters, and slight vision loss. On physical exam there is no trauma, visual acuity is decreased, pupillary reflexes are normal. Fundoscopic examination shows the presence of vitreous hemorrhage which is the leakage of blood into the surrounding clear vitreous, blocking light from reaching the retina. On what structure in the retina is incoming light normally focused?

A
  1. Blind spot
  2. Optic cup
    3. Macula
  3. Optic disc

  • The macula is also called the macula lutea for its yellowish pigmented appearance and offers the greatest visual acuity within the retina.
  • Light is focused onto the macula which contains a high concentration of cone cells and is located temporally from the optic disc.
  • The photoreceptor cells within the macula transmit light sensory information onto the bipolar cells and then to the ganglion cells which become the optic nerve.
  • The symptoms presented are common in retinal detachment which results in vision loss as the macula is displaced and the photoreceptor cells degenerate from ischemia.
70
Q

A 72-year-old man presents to the clinic for pre- operative evaluation. The patient says he is feeling well, remembers many years ago, was diagnosed with some joint condition, and was advised to take mediations for which he would have to do blood tests regularly. He was not interested in doing the regular blood tests and seeing a clinician regularly, hence he never took treatment for the disease. On examination has bilateral ulnar deviation and subluxation of the metacarpophalangeal (MCP) joints. Hand X-ray shows erosions of MCP and proximal interphalangeal (PIP) joints. Rheumatoid factor (RF) and anti-CCP antibodies are positive. Which of the following is the next best step in the management of this patient?

A
  1. Stress test
    2. Cervical spine radiographs
  2. Pulmonary function tests
  3. Chest radiograph

  • Rheumatoid arthritis is a disease of the synovium. At the spine, C1 and C2 are the only vertebral bodies that have synovium. Hence, the only vertebral bodies which RA involves are C1 and C2. When RA affects the synovium tissue between these C1 (atlas) and C2 (axis), it may result in abnormal movement of the atlas relative to the axis. This abnormal movement may result in injury to the spinal cord.
  • The involvement of the atlantoaxial joint generally occurs in patients with long-standing RA. These patients often have severe peripheral disease, joint deformities of the hands as a result of RA. Erosion might be present on radiographs.
  • Most of the time, these patients have not been adequately managed for RA. This may be due to a lack of regular follow up for RA due to nonadherence or other reasons.
  • When RA patients are about to undergo general anesthesia, it is best to have an X-ray of cervical spine to evaluate for atlantoaxial instability to prevent any damage to the spinal cord at the time of intubation.
71
Q

Lumbar puncture in a patient with suspected subarachnoid hemorrhage is most sensitive during what time period?

A
  1. Immediately after the rupture of aneurysm
  2. Within the first 2 hours
    3. At 12 hours
  3. After 48 hours

  • LP is often negative when done within the first 2 hours after the bleed.
  • LP is most sensitive at about 12 hours after symptoms occur.
  • Noncontrast CT is usually diagnostic and done before LP.
  • An LP is usually performed to assess for presence of RBCs and xanthochromia. Unlike a traumatic tap, the number of RBC in subarachnoid hemorrhage will remain elevated in all tubes. The fluid can also be spun down. Xanthochromia of the supernatant is usually seen in subarachnoid hemorrhage.
72
Q

A 16-year-old patient underwent placement of an external ventricular drain for the management of acute hydrocephalus secondary to tubercular meningitis. Two days later he developed a high-grade fever associated with chills and rigor. He had neck stiffness on clinical examination. The CSF culture showed the growth of Pseudomonas aeruginosa sensitive to amikacin only. The treating clinician planned to start intraventricular amikacin therapy. What is the safe daily dosage of amikacin that can be administered intraventricularly in the patient?

A
  1. 10 mg
    2. 30 mg
  2. 60 mg
  3. 100 mg

  • Intraventricular instillation of antibiotics is a valid option in treating patients with ventriculitis.
  • The dose of amikacin through the intraventricular route is 1/20th of the intravenous dose.
  • The average daily dose of amikacin through the intraventricular route is 30 mg. The maximum daily dose of amikacin should not exceed 50 mg.
  • The dosage of amikacin in excess of 30 mg daily has shown to be associated with a high incidence of adverse effects, including choroid plexitis and the risk of an intraventricular bleed.
73
Q

A patient with a history of obesity, known lumbar spondylosis, and chronic low back pain presents to your clinic. She has failed conservative management of her pain, including physical therapy, weight loss, diet modification, and oral medications. You suspect her pain is due to lumbar facet arthropathy. Diagnostic L3-L5 medial branch block with lidocaine results in the patient obtaining 80% pain relief that lasts for 3 hours. What is the most appropriate next step?

A
  1. Refer to a spine surgeon for lumbar fusion
  2. Proceed with medial branch radiofrequency ablation
    3. Repeat the medial branch block with a longer-acting local anesthetic
  3. Perform intra-articular facet injections using a mixture of lidocaine and betamethasone

  • There is a 25% to 40% false positive rate with medial branch blocks. Thus, a second block is necessary to establish a diagnosis of facetogenic pain.
  • If the patient’s pain is truly facetogenic, a second diagnostic block should produce pain relief for a longer duration than the lidocaine block. Bupivacaine is a longer-acting agent that is commonly utilized.
  • Since only lidocaine was used, the pain relief obtained should be transient.
  • The goal of medial branch radiofrequency ablation is to provide six or more months of relief. It can be repeated.
74
Q

A 40-year-old female presented with daily hemicranial headaches precipitated by bright light. She has a moderate disability with it. She has to take medicines every time to relieve the pain. In which grade of Migraine Disability Assessment Questionnaire (MIDAS), is this patient?

A
  1. Grade I
  2. Grade II
    3. Grade III
  3. Grade IV

  • Moderate disability comes under Grade III of MIDAS score.
  • Grade I shows little or no disability, grade II mild disability, and grade IV severe disability.
  • The need to take medicines everytime is also an indicator of moderate to severe disability.
  • This patient will require prophylactic treatment of migraine.
75
Q

A 67-year-old widowed male has a progressive headache for two weeks. For the past three days, he has been increasingly confused during his daily telephone conversations with his daughter who lives out of town. She called emergency medical services, and the patient is taken to the hospital. The patient is mildly confused, repeating himself often, but otherwise, alert without weakness. His lab workup is unremarkable including complete blood count, complete metabolic panel, urine analysis and chest x-ray. A computed tomography (CT) scan of the head identifies a 1.4 cm right subdural hematoma with mixed density. The patient’s daughter states two weeks ago the patient was very mentally sharp and never repeated himself. She states the would want definitive treatment with the lowest likelihood of recurrence. Which options provides the least likelihood of recurrence of the subdural hematoma for this patient?

A
  1. Dexamethasone and serial imaging
  2. Twist drill hole drainage of the subdural hematoma
  3. Two burr hole drainage of the subdural hematoma
    4. Craniotomy with drainage of the subdural hematoma

  • Surgery is generally recommended for subdural hematomas which are either symptomatic, 1 cm or more in greatest thickness, or causing 5 mm or more of midline shift.
  • A twist drill hole for subdural hematoma evacuation has the lowest operative complication rate and highest recurrence rate.
  • A burr hole evacuation of subdural hematoma has an intermediate operative complication rate and intermediate recurrence rate.
  • A craniotomy for subdural hematoma evacuation has higher operative complication rate than twist drill hole or burr hole evacuation but also has a lower recurrence rate than either two procedures.
76
Q

A 25-year-old man is brought in following a head- on motor vehicle accident. He is stable after the primary survey. On examination, he has severe mid-upper facial injuries. A head CT scan reveals complex facial fractures which are classified using the Markowitz-Manson system. Which of the following structures form the boney confluence that this system stages?

A
  1. Nasal, lacrimal, maxillary, and parietal bones
  2. Lacrimal, ethmoid, maxillary, and occipital bones
    3. Nasal, lacrimal, ethmoid, maxillary, and frontal bones
  3. Nasal, lacrimal, ethmoid, maxillary, and temporal bones

  • The nasoorbitoethmoid (NOE) is the confluence of the frontal sinus, ethmoid sinuses, anterior cranial fossa, forehead, orbits, and nasal bones.
  • The NOE encompasses the vertical and horizontal buttresses of the central face.
  • Nasoorbitoethmoid fractures are caused by high impact anterior force to the nose and then transmitted posteriorly towards the ethmoid. This can result in telecanthus, nasofrontal duct disruption, orbital injuries, and rhinorrhea.
  • A thorough understanding of NOE anatomy is critical to avoid complications and obtain an aesthetic surgical result.
77
Q

A 54-year-old female presented one year ago with chronic low back pain. Imaging at the time identified an L5-S1 left-sided annular disc tear. Six months ago, she had an acute exacerbation of the pain with pain radiating into the posterolateral left leg. New imaging identified a left L5-S1 disc herniation which was treated with microdiscectomy. She was doing well initially after surgery but now presents to the clinic with recurrent left-sided posterolateral leg pain. Current repeat imaging shows recurrent L5- S1 left-sided disc herniation, which is treated with repeat micro lumbar discectomy. Which of the following is most accurate regarding closing an annular fissure?

A

1. There is no convincing evidence for closing an annular fissure
2. Only close an annular fissure after the third disc herniation at the same location
3. An annular fissure should be closed whenever encountered
4. An annular fissure should only be closed in patients with a body mass index (BMI) greater than 30 kg/m^2

  • There is no convincing evidence for closing an annular fissure.
  • Many surgeons consider a fusion if there are three or more recurrent disc herniations at the same level although some would consider fusion after the first recurrence.
  • Some patients may need fusion with the first disc herniation depending on other radiologic and anatomic factors.
  • Many different devices have been developed for closing an annular fissure but robust data for their use is still pending.
78
Q

A 53-year-old male is being monitored in the emergency department after experiencing a tonic-clonic seizure a few hours ago. The patient is currently being treated with a combination of IV valproic acid/carbamazepine. The patient is awake and oriented and is complaining of a headache. His blood pressure is 125/75 mmHg, heart rate 95 beats/min, respiratory rate 18, and temperature 36.6 C (97.8 F). The patient’s history reveals that while he was having the seizure, he slid off the edge of his bed and hit his head on the corner of his nightstand. While taking the history, the emergency care provider observes the patient struggling to keep his eyes open and remain lucid. After a few minutes, the patient loses consciousness, and a code is called. The patient’s vitals at this point are blood pressure 144/52 mmHg, heart rate 48 beats/min, respiratory rate 9, and temperature 36.6 C (97.8 F). An emergent CT reveals epidural hemorrhage with associated fracture of the patient’s right temporal bone. The patient is rapidly transferred to the operating room for an emergent craniotomy. What drug is may be less effective than usual when providing anesthesia care to this patient?

A
  1. Dantrolene
  2. Lorazepam
    3. Rocuronium
  3. Mannitol

  • Conditions that can cause resistance to non-depolarizing neuromuscular blockers include cerebral palsy, burn injuries, hemiplegia, peripheral nerve injuries, chronic infections of botulism, and tetany.
  • Rocuronium dosing for intubation is IV 0.45 to 0.90 mg/kg and maintenance with 0.15 mg/kg boluses.
  • In this patient, non-depolarizing neuromuscular blockers may not be effective because the patient was administered a combination of two anticonvulsants (valproic acid, carbamazepine). Anticonvulsants can cause increased resistance to blockade activity.
  • Other agents that cause resistance are the anticholinesterase inhibitors neostigmine and pyridostigmine.
79
Q

A 65-year-old male presented in the emergency department with the complaint of loss of sensation in both lower limbs following a firearm injury to his lower back. On examination, power is 2/5 in both lower limbs. There is a poor anal sphincter tone on the digital rectal examination (DRE). The clinician on duty is confused about making a diagnosis between conus medullaris and cauda equina syndromes. Which of the following is an important difference in the presentation of conus medullaris and cauda equina syndromes?

A

1. Conus medullaris syndrome presents with upper motor neuron signs like spasticity and hyperreflexia
2. Conus medullaris syndrome usually presents with unilateral lower extremity weakness
3. Cauda equina only, and not conus medullaris results in saddle anesthesia
4. Conus medullaris and cauda equina syndromes always have the same exact presentation

  • Because CMS always involves the terminal end of the spinal cord at around T12/L1, while CES can involve the conus or only involve the L1-L5 nerve roots, it is much more likely to present with upper motor neuron signs. CES usually presents with lower motor neuron signs only, such a hypotonia, hyporeflexia due to the termination of the upper motor neurons synapsing into the cord above the cauda equina nerve roots.
  • Saddle anesthesia is neither sensitive not specific for the CES/CMS syndromes and does not carry a high positive predictive value for the diagnosis.
  • The bladder receives innervation from both the parasympathetic and sympathetic systems and loss of either or both systems can lead to urinary retention in CES/CMS.
  • Surgical decompression is commonly done through the posterior midline approach to the lumbar spine.
80
Q

A 44-year-old man with a prior history of glioblastoma multiforme treated with surgical excision and radiation therapy presents with symptoms of memory loss, headaches, drowsiness, and periods of blank staring. These periods usually last 2 to 3 minutes, and he is unresponsive. The symptoms started about six months after completion of the radiation therapy. An MRI brain shows a defect in the right temporal lobe with a palisading pattern of necrosis, and there is an absence of T2 flair involving the surrounding white matter. What is the most appropriate next step in management?

A

1. Recommend surgical biopsy of the area for definitive diagnosis
2. CT angiography to assess for concomitant cerebral vascular disease
3. Starting hyperbaric oxygen therapy to treat brain tissue radiation necrosis
4. Recommend referral to a radiation oncologist to consider further palliative radiation therapy

  • This patient’s MRI is compatible with tumor recurrence; glioblastoma multiforme commonly shows a palisading pattern.
  • When symptoms plus an MRI is not diagnostic for radiation necrosis, it is best to biopsy for definitive diagnosis and treatment.
  • The underlying pathologic process for radiation necrosis is ischemia due to endovascular breakdown.
  • Subjecting the patient to inappropriate therapeutic trials of hyperbaric oxygen therapy would delay definitive diagnosis and treatment. Further radiation therapy could cause worsening of the patient’s condition if the underlying problem is radiation necrosis.
81
Q

A 25-year-old has had refractory migraine attacks for the last two years. The pain starts in the left temporal region and spreads to the ipsilateral hemicrania. Botulinum toxin injection in the left temporal region caused the cessation of her pain. The effect, however, lasted only for three months despite three repeated injections. It has now affected the activities of her daily living as well. Which of the following is the next best step in the management of the patient?

A
  1. Continuing botulinum toxin injection every month
    2. Lysis of left zygomaticotemporal nerve
  2. Lysis of left supratrochlear nerve
  3. Lysis of auriculotemporal nerve

  • This patient has a refractory migraine. The trigger site is at the left temporal region, which has been confirmed by botulinum toxin. Repeat injections of botulinum toxin lead to the development of antibodies to the toxin, which leads to the development of resistance, decreasing its effects.
  • The zygomaticotemporal nerve is the sensory supply to the temporal region. Lysis of this nerve can lead to relief of her symptoms.
  • The incision is made at the superior temporal line.
  • It can be performed through either open or endoscopic routes.
82
Q

A 46-year-old patient presents with bilateral had pain for the past month described as a “pins and needles” sensation involving all five digits on both hands and associated decreased grip strength bilaterally. On further questioning, she admits to aching neck pain over the same period, though she cannot recall any trauma or other inciting events. On exam, she is found to have a positive Spurling test. What is the next most appropriate step in management?

A

1. Magnetic resonance imaging (MRI) of the cervical spine to evaluate for cervical pathology
2. X-ray of the cervical spine, including lateral flexion and extension views
3. X-ray of bilateral hands and wrists
4. Surgery referral for evaluation and treatment of carpal tunnel syndrome

  • Due to its high specificity, a positive Spurling test is highly indicative of cervical radiculopathy, and further imaging of the cervical spine is often warranted. To perform the Spurling test, the examiner turns the patient’s head to the affected side while extending and applying downward pressure to the top of the patient’s head. A positive sign is if the pain starts in the neck and radiates in the direction of the corresponding ipsilateral dermatome.
  • In patients with cervical radiculopathy, MRI is the imaging study of choice. X-ray is an excellent first choice in the case of trauma, but in non-traumatic cervical radiculopathy, it is rarely diagnostic.
  • In patients with atypical presentation of carpal tunnel syndrome such as neck pain or hand pain not limited to median nerve distribution, cervical pathology should be considered.
  • In patients with moderate to severe carpal tunnel syndrome, a surgical referral is appropriate, but cervical radiculopathy should be ruled out in patients with atypical presentation.
83
Q

A 45-year-old patient on antitubercular therapy (ATT) for pulmonary tuberculosis is brought to a peripheral hospital with a history of progressive headache for two days, followed by multiple episodes of vomiting and altered sensorium for one day. On arrival, his Glasgow Coma Scale (GCS) score is E2V2M3, with both pupils equal but sluggishly reactive. His fundoscopic examination reveals the presence of bilateral papilledema. An urgent computed tomogram (CT) scan of his brain with contrast reveals the presence of communicating hydrocephalus with thick basal exudates. He undergoes prompt external ventricular drain (EVD) placement. He shows improvement in his motor score to M5 within 24 hours. What would be the most rational next step of management for this patient?

A
  1. Continue with EVD drainage and slowly plan to withdraw it once the patient stabilizes
  2. Plan for endoscopic third ventriculostomy (ETV)
    3. Plan for eventual ventriculoperitoneal (VP) shunting
  3. Discharge the patient once he stabilizes on the ATT regime

  • Early VP shunting is justified in post TB meningitis communicating hydrocephalus who show significant recovery following EVD placement.
  • Early shunt minimizes the risk of infection and meningitis following prolonged days of EVD placement.
  • There is also no added risk of the shunt being obstructed and nonfunctioning since the tubercular bacilli cannot colonize the shunt tube since they don’t form the slime layer.
  • ETV shows a low success rate in communicating hydrocephalus secondary to tuberculosis. It is also not possible in all centers. However, it has been suggested in patients with chronic illness and VP shunt is suggested in the acute phase of illness. Only ATT therapy would not counteract the hydrocephalus part in TB meningitis. They invariably require permanent CSF diversion
84
Q

A 14-year-old boy with a known history of Klippel- Feil syndrome presents to his family practice doctor with ongoing falls and new onset weakness in the left upper extremity. What is the best imaging modality to further evaluate the symptoms?

A
  1. MRI with contrast
    2. MRI without contrast
  2. CT without contrast
  3. Plain x-rays

  • MRI without contrast is necessary to see the spinal cord, ligaments, soft tissue, and disc spaces in this patient.
  • X-ray is a good imaging modality for initial diagnosis of Klippel- Feil syndrome but is not adequate in this case.
  • CT scan is an appropriate imaging modality for the evaluation of bony fusion for possible operative planning.
  • CT scan without contrast is a good imaging platform to evaluate cervical spine fracture in this population as well.
85
Q

A 70-year-old female with a history of scoliosis and prior lumbar fusion presents to the clinic complaining of chronic back pain. On physical exam, she has a crouched gait with compensatory knee flexion. She exhibits no neurologic deficits and does not complain of pain in a regional pattern. Which of the following is the mainstay of spinal alignment imaging?

A

1. 36-inch standing films
2. CT scan
3. EOS X-ray system
4. MRI spine

  • For imaging, 36-inch standing films are the mainstay of spinal alignment imaging.
  • A standard 36-inch, two-view x-ray is crucial to evaluate the global spine and lumbopelvic relationships. Sagittal alignment parameters such as pelvic tilt (PT) and sagittal vertical axis (SVA) are measured on these images, and preoperative planning requires these values.
  • New biplanar x-ray with 2D to 3D reconstruction systems allow standing evaluation of the global spine and pelvis, which provides the benefits of both plain film and CT scan. The main advantage over traditional CT is the lower dose of radiation, which may be 800 to 1000 times less. However, this is a new technology and is only used at select centers at the current time.
  • MRI is useful for the examination of soft tissue deformities and neural elements. This patient does not exhibit any signs of stenosis or radiculopathy. It is important to remember the CT/MRI modalities are recombinant imaging platforms, not standing.
86
Q

A 60-year-old male has a sudden onset of extreme dizziness, nausea, and vomiting while at work. He is taken to the hospital for a stroke workup. On exam, he opens his eyes to voice, is oriented to conversation, follows commands with all extremities and has profound bilateral dysmetria of the upper and lower extremities. His blood pressure is 180/100 mmHg, heart rate of 102 bpm, and oxygen saturation of 98% on room air. Imaging reveals a cerebellar hemorrhage of the bilateral cerebellar hemispheres with 40 mL of estimated blood volume and effacement of the fourth ventricle with lateral ventricular enlargement. There does not appear to be any blood in the ventricular system. What is his 30-day mortality based on the intracerebral hemorrhage (ICH) score?

A

1. 26%
2. 72%
3. 97%
4. 100%

  • The intracerebral hemorrhage (ICH) score predicts 30-day mortality based on patient with non-traumatic intracerebral hemorrhage.
  • The score is based on Glasgow coma score (GCS), age, volume of hemorrhage, intraventricular hemorrhage, and infratentorial origin of hemorrhage.
  • The point distribution is GCS 13 to 15 is 0 points, GCS 5 to 12 is 1 point, GCS 3 to 4 is 2 points; age 80 or greater is 1 point, ICH volume 30 mL or more is 1 point, intraventricular hemorrhage is 1 point, and infratentorial origin of hemorrhage is 1 point.
  • 30-day mortality is 0% for 0 points, 13% for 1 point, 26% for 2 points, 72% for 3 points, 97% for 4 points, 100% for 5 points, and assumed to be 100% for 6 points.
87
Q

A 57-year-old woman with past medical history significant for depression and hypertension presents with intractable lower back pain with radiculopathy to her left anterior thigh and feet. Her depression and hypertension are appropriately managed by her primary care provider. However, her low back pain continues to persist despite conservative management with physical therapy, use of neuropathic medications, and lumbar fusion of L3 – L5. What is her most likely diagnosis?

A
  1. Major depression
  2. Fibromyalgia
  3. Hypertensive emergency
    4. Failed back surgery syndrome

  • This patient presents with low back pain despite surgical intervention. Her presentation is concerning for failed back surgery syndrome (FBSS). Numerous factors are associated with the development of FBSS despite surgical intervention. This includes history of multiple spine or back surgeries, psychiatric comorbidities such as depression, and the presence of ongoing litigation or worker’s compensations claims. This patient presents with persistent low back pain despite conservative and surgical intervention. Her persistent low back pain despite multiple prior back surgeries suggests a clinical diagnosis of failed back surgery syndrome.
  • Patients with fibromyalgia present with multiple tender areas that are may respond to neuropathic medications. The history of persistent low back pain despite surgical intervention and conservative management suggests failed back surgery syndrome.
  • This patient’s hypertension is currently well controlled by her primary care physician. It is very unlikely that low back pain with radiculopathy could be explained by a hypertensive emergency.
  • Failed back surgery syndrome (FBSS) is a very difficult condition to treat. FBSS should be high on the differential for patients with persistent low back pain after surgical and conservative interventions. Appropriate diagnosis can result in appropriate treatment with modalities like spinal cord stimulator implantation. Spinal cord stimulator implants have been shown to reduce low back pain among patients with FBSS dramatically
88
Q

A 50-year-old male patient is seen in the trauma bay after a high-speed unhelmeted motorcycle accident in which he was struck in the neck with a wire fence in a “clothes-line” injury. The patient is found on computed tomography to have fractures of his second and third cervical vertebrae with extension of the fracture at the third vertebrae through the transverse foramen. One week later, while the patient is recovering from his traumatic injuries in the intensive care unit, the patient states that he is unable to see out of either eye. What test would have been most prudent to obtain at the patient’s initial presentation to diagnose this catastrophic complication?

A
  1. Magnetic resonance imaging of the neck
    2. Computed tomography with angiography of the head and neck
  2. Carotid duplex ultrasonography
  3. Magnetic resonance angiography of the head

  • Blunt cerebrovascular injury (BCVI) is very common in patients who are subjected to high energy transfers during an accident. The Denver criteria was established to define screening parameters for patients who are susceptible to BCVI and who should undergo formal testing for diagnosis and treatment.
  • Even with appropriate screening parameters, as suggested by the Denver or modified-Memphis criteria, 20% of patients are missed who have clinically significant BCVI.
  • Multi-slice, high-resolution computed tomography with angiography of the head and neck has become as close to the gold standard of formal four-vessel cerebrovascular angiogram as possible. It is considered the test of choice to evaluate patients for blunt cerebrovascular injury. Its downsides are potential additional radiation dosage (especially in children) as well as an increase in dye load which can lead to contrast nephropathy. Furthermore, for low-grade injuries, Biffl grade I, it has a high false-positive rate, which approaches 48%.
  • Magnetic resonance angiography is a useful alternative to evaluate for blunt cerebrovascular injury, especially in patients with acute kidney injury as it doesn’t utilize IV contrast. However, it has less sensitivity than formal computed tomography with angiography. Furthermore, it does have additional downsides such as it takes longer to perform, the patient must remain still (difficult in children or intoxicated/uncooperative patients), removes a potentially sick patient from a monitored setting. To evaluate for BCVI, MRA should be done for the head and neck and not just for the head as the answer states. Carotid duplex angiography is able to evaluate the carotid arteries well but has very low sensitivity evaluating the posterior circulation consisting of the vertebral arteries which are enclosed within a bony canal. It is not considered adequate as a screening tool to evaluate for BCVI.
89
Q

A 65-year-old male is 3 years status post intrathecal pump placement with morphine for chronic cancer pain with good pain control. Recently he reports worsening of the pain and some weakness in his left lower extremity compared to his baseline. After interrogating his intrathecal pump, you notice a 20% difference between expected and measured residual volume in the pump. What is your immediate next step in management?

A
  1. Increase patient’s oral opioid dose regimen
  2. Immediately inject contrast through side port under fluoroscopy to assess intrathecal catheter
  3. STAT Thoracolumbar x-ray
    4. STAT Thoracolumbar MRI

  • An intrathecal granuloma is a non-infectious mass of fibroblasts, collagen and a few inflammatory cells that usually arise from the arachnoid layer of the meninges as a result of an inflammatory reaction near the site of the catheter tip.
  • Risk of granuloma development appears to correlate directly with duration if IT therapy, daily dose and rate of drug titration.
  • Patients who present with increasing pain and change in their neurologic function should be evaluated for granuloma formation.
  • If granuloma is suspected the diagnosis should be confirmed with an MRI with or without contrast to assess the catheter tip.
90
Q

A 65-year-old female is referred to physical therapy for neck pain. She states she has been experiencing vertigo with diplopia as well as a persistent headache. Diplopia and vertigo are reproducible when she tilts her head up. What is the best next step in diagnosis?

A
  1. Duplex ultrasound
    2. MR angiogram or CT angiogram
  2. Work up for hypercoagulable states
  3. MRI cervical spine

  • CT angiogram or contrast MR angiogram are best in visualizing the vertebrobasilar system to define stenosis or occlusions. The studies also can be performed when the patient is asked to extend his or her neck to reproduce the symptoms.
  • Duplex ultrasound may also be used to visualize the vertebral artery, but the images generally are not good enough for the diagnosis.
  • MRI is a better imaging modality for brainstem and posterior fossa problems, especially acute brainstem infarction with diffusion-weighted imaging. However, MRI cervical spine will not show stenosis or occlusion of the vertebral arteries which are the presenting symptoms of this patient.
  • If a patient with vertebrobasilar insufficiency is younger than 45 years of age, further workup is indicated to rule out a cardioembolic cause, hypercoagulable states, vertebral dissection, and fibromuscular dysplasia.
91
Q

A 35-year-old male patient is being managed for post-meningitic hydrocephalus with the placement of an extraventricular drain (EVD). The patient gradually shows clinical improvement, and repeat CT images did not show any progression of the hydrocephalus. The patient is thus planned for EVD removal. Which of the following best describes the ideal method for the safe removal of the EVD in such patients?

A
  1. Remove the EVD, place a stitch at the exit wound and watch for clinical deterioration
  2. Remove the EVD, place a stitch at the exit wound, and perform alternate day CT head to watch for progression of hydrocephalus 3. Remove the EVD, place a stitch at the exit wound, and perform daily optic nerve sheath diameter monitoring
    4. Gradual challenging of the EVD up to 15 cm above the tragus for 24 hours, followed by removal and stitching of the exit wound

  • Removal of EVD in cases of hydrocephalus sometimes poses a risk of progression of the hydrocephalus and clinical deterioration following its removal.
  • It has been found that a gradual rise in the EVD draining height without closing the drain system (training the EVD) would prevent the risk of an inadvertent rise in ICP by the egress of CSF only at high-pressure points.
  • Clamping the EVD is an option, but there can be sudden deterioration due to the progression of hydrocephalus and can be missed or diagnosed late based on clinical examination only.
  • Repeat images to assess any progression in hydrocephalus are valid only in cases of chronic and slowly progressive hydrocephalus. Sudden deterioration following acute hydrocephalus requires either reinserting the EVD or opening the clamped drainage system.
92
Q

A 42-year-old woman presents to the clinic to assess options for her cancer-related pain. The patient currently works as a museum curator and has a history of von Willebrand disease, hypertension, diabetes mellitus, liver failure with severe coagulopathy, and metastatic breast cancer. Her vital signs are all within normal reference ranges. She rates her pain as 6 out of 10 in her pelvis with a continuous infusion of opioids and oral gabapentin, and the pain in her right leg is making it difficult for her to work. She is interested in definitive, permanent options to palliate her pain. Which of the following is the most appropriate approach to this patient?

A
  1. Open bilateral cordotomy; if hypertension can be controlled, proceed with the procedure as the patient wants definitive palliation of her pain
  2. Right unilateral percutaneous cervical cordotomy; even if blood glucose cannot be adequately controlled, proceed with the procedure to target the patient’s leg pain
  3. Midline myelotomy; if the patient’s pain remains at level 3 for 2- weeks, proceed with the procedure to address the patient’s pelvic pain
    4. The patient is not a surgical candidate for the intervention which would meet her needs

  • Cordotomy leads to the nociceptive pathways in the lateral spinothalamic tract anterolateral column being destroyed in the cervical spinal cord at the level of C1-C2.
  • Percutaneous cordotomy is contraindicated in patients with a coagulation disorder, such as patients with liver failure.
  • Patients must be at level 3 of pain on the World Health Organization (WHO) cancer ladder with a prognosis of under six months to be a candidate for cordotomy.
  • In otherwise healthy patients without contraindications, the risk of severe complications with unilateral cordotomy is low.
93
Q

A 2-month-old female infant is brought in by her grandmother due to increased crying, vomiting, and decreased oral intake. As per her grandmother, the patient was in her usual state of health 2 days prior when she last saw her. Vital signs are stable, and the patient is afebrile. On physical examination, the infant is irritable with a high pitched cry. The anterior fontanelle is bulging. The rest of the physical has symmetric chest expansion, clear breath sounds, and no adventitious sounds. The abdomen is soft and non-tender. The skin is clear with no rashes or lesions. Labs and imaging are performed. The chest x-ray shows lucencies in the posterior ribs. What is the diagnosis?

A
  1. Meningitis
    2. Inflicted traumatic brain injury
  2. Pneumonia
  3. Acute bronchiolitis

  • The findings of an irritability high-pitched cry and bulging fontanelle are suggestive of inflicted traumatic brain injury, also known as abusive head trauma, non-accidental head trauma, or shaken-baby-syndrome.
  • The lucencies of the posterior ribs are suggestive of posterior rib fractures. Posterior rib fractures are suggestive of child abuse.
  • The inflicted traumatic brain injury occurs due to the rapid acceleration-deceleration caused by shaking and direct impact to the head causing tearing of the bridging veins.
  • Inflicted traumatic brain injury can be manifested with nonspecific symptoms such as fussiness, altered mental status, and vomiting. Head CT scan is the next diagnostic study.
94
Q

A 16-year-old female is brought to the clinic for evaluation of multiple recurrent burn wounds sustained while cooking food. Her neurological examination reveals the presence of dissociative sensory loss along her bilateral upper arms and anterior part of the nape of her neck and upper chest. Further radiological workup reveals that the tip of the odontoid process, anterior arch of atlas, and clivus in unison were above the Chamberlain line but below McRae and Wackenheim lines. She also has cerebellar tonsils well below the foramen magnum. Based on these findings, which of the following is the most rational approach for managing the patient?

A
  1. Trans-oral decompression and posterior fusion
  2. Posterior fusion only
  3. Excision of the cerebellar tonsils and duraplasty
    4. Posterior fossa and foramen magnum decompression

  • The case depicts Group B basilar invagination with Chiari malformation.
  • The current recommendation is posterior fossa and foramen magnum decompression only. Since dura is expansile there is no need for performing duroplasty.
  • Since basilar invagination and resulting crowing of the posterior fossa predisposes to Chiari malformation, standalone surgical decompression is justified since there is a reversal in the Chiari process following the same.
  • Transoral decompression is now not advocated even for Group A Basilar Invagination. Duroplasty predisposes to a pseudomeningocele formation as well as added risk of CSF leak and meningitis.
95
Q

A 62-year-old female patient is seen in the clinic for her annual follow-up visit. She has a past medical history of hypertension, hyperlipidemia, and depression. Her current medications include aspirin, simvastatin, lisinopril, and fluoxetine. She was diagnosed with celiac disease at age 28 and follows a gluten-free diet. She notes that she has felt increasingly fatigued over the last few months. She has also noticed a new-onset of numbness and tingling in her lower extremities, as well as problems with balance. Her physical exam was significant for a noted decrease in sensation to both pinprick and light touch of her bilateral lower feet. She was also found to have a positive Romberg test. A complete blood count (CBC) reveals a decrease in hemoglobin, a decrease in hematocrit, and an increased mean corpuscular volume (MCV). The physician suspects a vitamin deficiency as the cause of this patients neurological problems. Which of the following structures is most likely affected by this patient’s suspected vitamin deficiency?

A
  1. Cerebellum
    2. Posterior columns of the spinal cord
  2. Lateral columns of the spinal cord
  3. Anterior corticospinal tract

  • Deficiency of vitamin B12 causes damage to both the posterior and lateral columns of the spinal cord.
  • There is gliosis, demyelination, and vacuolar degeneration predominantly of the upper thoracic and lower cervical regions.
  • There is a subsequent axonal loss.
  • A positive Romberg indicates impaired position sense, carried by the posterior columns.
96
Q

A 76-year-old female presents for preoperative evaluation for carotid endarterectomy. She has well-controlled diabetes mellitus, hypertension, and hyperlipidemia. Her medications include aspirin, atorvastatin, and chlorthalidone. She has no cardiac symptoms or history. The only physical finding is a left carotid bruit. The ECG is normal except for rare premature ventricular contractions. Laboratories show normal electrolytes, hemoglobin A1c, liver function tests, renal function, and lipid profile. Which of the following is the expected risk of life-threatening complications in the patient?

A
  1. 0.4 percent
    2. 0.9 percent
  2. 6.6 percent
  3. 11 percent

  • As per the revised cardiac risk index, this patient has only one risk factor for possible life-threatening complications.
  • The planned surgical intervention (carotid endarterectomy) is the only risk variable in the patient.
  • This one point gives a risk of 0.9% for any major complications in the patient.
  • 2 points have a risk of 6.6% whereas 3 or more points have a risk of 11%.
97
Q

A 65-year-old man with a history of bitemporal hemianopsia is undergoing a surgical evaluation. The patient describes severe thirst and seems lethargic and disoriented. MRI of the head and neck reveals a craniopharyngioma. Transsphenoidal hypophysectomy is performed without apparent complications. Two hours after surgery, he develops polyuria (375 mL/h) and hypernatremia (155 mEq/L). Which of the following is the next best step in the management of this patient?

A
  1. Indomethacin
  2. Amiloride
    3. Intravenous desmopressin
  3. Hydrochlorothiazide

  • insipidus (DI) is a common complication following pituitary surgery. DI is a disease process that results in either decreased release of or response to antidiuretic hormone (ADH, also known as vasopressin or AVP), which can cause electrolyte
  • Most cases of diabetes insipidus following transsphenoidal hypophysectomy are transient. About 20% of patients who undergo neurosurgery will present with diabetes insipidus.
  • Treatment is with fluid replacement and desmopressin. The most common findings in patients with diabetes insipidus are polydipsia, polyuria, and nocturia.
  • Risk factors for developing diabetes insipidus due to transsphenoidal hypophysectomy include craniopharyngioma or extensive intraoperative handling of the pituitary gland. Hydrochlorothiazide is used for resistant DI.
98
Q

A 58-year-old male presented twelve months ago with low back pain. Workup at the time revealed an annular disc tear at L4-L5 level without disc protrusion. He was prescribed NSAIDs as well as low-impact physical therapy. This failed to relieve his symptoms, and he underwent three rounds of transforaminal steroid injections at the affected level with temporary relief only. He presents to the clinic with continued pain and a new magnetic resonance image showing some mild T1 contrast enhancement at the annular disc tear without disc protrusion or foraminal stenosis. No relative instability is noted on dynamic flexion-extension x-rays. What is the most appropriate surgical treatment modality for this patient?

A
  1. Foraminotomy at the affected level
  2. Lumbar decompression at the affected level
  3. Lumbar decompression and instrumented fusion at the affected level
    4. Resection of granulation tissue at the affected level

  • For patients who fail conservative treatment, resection of granulation tissue at a chronic annular disc tear can be considered.
  • Granulation tissue of an annular disc tear shows some contrast enhancement on T1 MRI sequencing.
  • First line treatment for symptomatic annular disc tear is non- steroidal anti-inflammatory medications and low-impact physical therapy.
  • Transforaminal lumbar steroid injections can be considered in patients with annular disc tears who fail non-steroidal anti- inflammatory medications and low-impact physical therapy.
99
Q

A 17-year-old obese male football lineman presents to your outpatient clinic complaining of recurrent, dull, achy low back pain which is worse with activity. You suspect spondylolysis. Which physical exam maneuver would be most useful to test for this condition?

A
  1. Straight leg raise
  2. Thompson test
  3. Spurling test
    4. Stork test

  • Spondylolysis usually remains asymptomatic with only 10% of affected individuals manifesting symptoms.
  • For symptomatic spondylolysis, patients’ typically present with an insidious, recurrent, achy low back pain which radiates to the buttocks and posterior thighs and is exacerbated by lumbar hyperextension activities.
  • On physical exam patients’ with spondylolysis will commonly demonstrate increased lumbar lordosis, tight hamstrings, and tenderness to palpation overlying the pars interarticularis fracture site.
  • The stork test involves the patient performing a single leg hyperextension and rotation of the spine and if this maneuver reproduces the patients pain it is considered diagnostic of spondylolysis until proven otherwise.
100
Q

An 80-year-old male presents to the clinic with a 10-year history of chronic pain in the lower back region when walking. Recently, he had an episode of fecal incontinence and urinary incontinence. His past medical history reveals arthritis, diabetes mellitus, hyperlipidemia, and hypertension. An MRI (magnetic resonance imaging) of the spine reveals spinal stenosis at the levels of L3 and L4. A decision is made to correct this with decompressive laminectomy. What are the layers from the outermost to the innermost penetrated by the surgeon during the procedure?

A

1. Skin- superficial fascia- thoracolumbar fascia- lamina portion of the L3/L4 vertebrae
2. Skin- superficial fascia- transverse process of the L3/L4 vertebrae
3. Skin- superficial fascia- posterior longitudinal ligament- lamina of the L3/L4 vertebrae
4. Skin- superficial fascia- thoracolumbar fascia- posterior longitudinal ligament- the spinous process of the L3/L4 vertebrae

  • The outermost layers involve the skin and superficial fascia. The thoracolumbar fascia is a deep investing fascia surrounding the posterior thorax and abdomen. This fascia of the lumbar spine assists in supporting the vertebral column when it is in a flexed position. The vertebral bodies can be found deep to the thoracolumbar fascia. This must be penetrated to reach the lamina of the vertebral bodies.
  • The transverse process of the vertebral bodies is not involved in the laminectomy procedure. It is the lamina portion of the vertebral body that is removed to allow space for the nerves to pass without compression.
  • The posterior longitudinal ligament is deep to the spinous process, transverse process, and lamina parts of the vertebral bodies. The laminectomy procedure would not require exposure of the posterior longitudinal ligament.
  • The spinous process of the lumbar vertebral bodies is superficial to the posterior longitudinal ligament. It also should be noted that the spinous process of the vertebral body is not involved in a laminectomy.